Add data
Browse filesThis view is limited to 50 files because it contains too many changes.
See raw diff
- README.md +102 -3
- processed_dataset/calculation/0001.json +8 -0
- processed_dataset/calculation/0002.json +8 -0
- processed_dataset/calculation/0003.json +8 -0
- processed_dataset/calculation/0004.json +8 -0
- processed_dataset/calculation/0005.json +8 -0
- processed_dataset/calculation/0006.json +10 -0
- processed_dataset/calculation/0007.json +8 -0
- processed_dataset/calculation/0008.json +8 -0
- processed_dataset/calculation/0009.json +10 -0
- processed_dataset/calculation/0010.json +8 -0
- processed_dataset/calculation/0011.json +8 -0
- processed_dataset/calculation/0012.json +8 -0
- processed_dataset/calculation/0013.json +8 -0
- processed_dataset/calculation/0014.json +8 -0
- processed_dataset/calculation/0015.json +8 -0
- processed_dataset/calculation/0016.json +8 -0
- processed_dataset/calculation/0017.json +8 -0
- processed_dataset/calculation/0018.json +8 -0
- processed_dataset/calculation/0019.json +8 -0
- processed_dataset/calculation/0020.json +8 -0
- processed_dataset/calculation/0021.json +8 -0
- processed_dataset/calculation/0022.json +8 -0
- processed_dataset/calculation/0023.json +8 -0
- processed_dataset/calculation/0024.json +8 -0
- processed_dataset/calculation/0025.json +8 -0
- processed_dataset/calculation/0026.json +8 -0
- processed_dataset/calculation/0027.json +8 -0
- processed_dataset/calculation/0028.json +8 -0
- processed_dataset/calculation/0029.json +8 -0
- processed_dataset/calculation/0030.json +8 -0
- processed_dataset/calculation/0031.json +8 -0
- processed_dataset/calculation/0032.json +8 -0
- processed_dataset/calculation/0033.json +8 -0
- processed_dataset/calculation/0034.json +8 -0
- processed_dataset/calculation/0035.json +8 -0
- processed_dataset/calculation/0036.json +8 -0
- processed_dataset/calculation/0037.json +10 -0
- processed_dataset/calculation/0038.json +8 -0
- processed_dataset/calculation/0039.json +8 -0
- processed_dataset/calculation/0040.json +8 -0
- processed_dataset/calculation/0041.json +8 -0
- processed_dataset/calculation/0042.json +8 -0
- processed_dataset/calculation/0043.json +8 -0
- processed_dataset/calculation/0044.json +8 -0
- processed_dataset/calculation/0045.json +8 -0
- processed_dataset/calculation/0046.json +8 -0
- processed_dataset/calculation/0047.json +8 -0
- processed_dataset/calculation/0048.json +8 -0
- processed_dataset/calculation/0049.json +8 -0
README.md
CHANGED
@@ -1,3 +1,102 @@
|
|
1 |
-
|
2 |
-
|
3 |
-
|
|
|
|
|
|
|
|
|
|
|
|
|
|
|
|
|
|
|
|
|
|
|
|
|
|
|
|
|
|
|
|
|
|
|
|
|
|
|
|
|
|
|
|
|
|
|
|
|
|
|
|
|
|
|
|
|
|
|
|
|
|
|
|
|
|
|
|
|
|
|
|
|
|
|
|
|
|
|
|
|
|
|
|
|
|
|
|
|
|
|
|
|
|
|
|
|
|
|
|
|
|
|
|
|
|
|
|
|
|
|
|
|
|
|
|
|
|
|
|
|
|
|
|
|
|
|
|
|
|
|
|
|
|
|
|
|
|
|
|
|
|
|
|
|
|
|
|
|
|
|
|
|
|
|
|
|
|
|
|
|
|
|
|
|
|
|
|
|
|
|
|
|
|
|
|
|
|
|
|
|
|
|
|
|
|
|
|
|
|
|
|
|
|
|
|
|
|
|
|
1 |
+
# BlueMO
|
2 |
+
|
3 |
+
## BlueMO: A Comprehensive Collection of Challenging Mathematical Olympiad Problems from the Little Blue Book Series
|
4 |
+
|
5 |
+
**BlueMO** is a comprehensive and challenging dataset comprising mathematical olympiad problems paired with detailed solutions, meticulously curated from the esteemed "Little Blue Book" (小蓝书) series (Second Edition)—a vital resource for Chinese students training for national and international olympiad math competitions.
|
6 |
+
|
7 |
+
Designed to advance and assess sophisticated reasoning in LLMs, this dataset serves as a benchmark or training resource for high-level problem-solving in AI.
|
8 |
+
|
9 |
+
## Introduction for "Little Blue Book" (小蓝书)
|
10 |
+
|
11 |
+
The "Little Blue Book" (小蓝书) series, published by East China Normal University Press, is a cornerstone resource for students striving to master mathematical olympiads. Renowned for its depth, challenging problems, and elegant solutions, the series spans critical domains—Sets, Trigonometric, Geometry, Number Theory, Graph Theory, Extremal Combinatorics—providing rigorous training for olympiad competitions.
|
12 |
+
|
13 |
+
## About Dataset
|
14 |
+
|
15 |
+
BlueMO encompasses a total of 14 volumes from the third edition of the "Little Blue Book" series, covering a wide range of mathematical topics for both middle and high school levels.
|
16 |
+
|
17 |
+
The dataset is structured as follows:
|
18 |
+
|
19 |
+
**High School Collection:**
|
20 |
+
|
21 |
+
* 小蓝书高中卷1 集合 - Little Blue Book High School Vol.1: Sets
|
22 |
+
|
23 |
+
* 小蓝书高中卷2 函数与函数方程 - Little Blue Book High School Vol.2: Functions & Functional Equations
|
24 |
+
|
25 |
+
* 小蓝书高中卷3 三角函数 - Little Blue Book High School Vol.3: Trigonometric Functions
|
26 |
+
|
27 |
+
* 小蓝书高中卷4 平均值不等式与柯西不等式 - Little Blue Book High School Vol.4: Mean Value & Cauchy Inequalities
|
28 |
+
|
29 |
+
* 小蓝书高中卷5 不等式的解题方法与技巧 - Little Blue Book High School Vol.5: Methods & Techniques for Solving Inequalities
|
30 |
+
|
31 |
+
* 小蓝书高中卷6 数列与数学归纳法 - Little Blue Book High School Vol.6: Sequences & Mathematical Induction
|
32 |
+
|
33 |
+
* 小蓝书高中卷7 平面几何 - Little Blue Book High School Vol.7: Plane Geometry
|
34 |
+
|
35 |
+
* 小蓝书高中卷8 复数与向量 - Little Blue Book High School Vol.8: Complex Numbers & Vectors
|
36 |
+
|
37 |
+
* 小蓝书高中卷9 几何不等式 - Little Blue Book High School Vol.9: Geometric Inequalities
|
38 |
+
|
39 |
+
* 小蓝书高中卷10 数论 - Little Blue Book High School Vol.10: Number Theory
|
40 |
+
|
41 |
+
* 小蓝书高中卷11 组合数学 - Little Blue Book High School Vol.11: Combinatorics
|
42 |
+
|
43 |
+
* 小蓝书高中卷12 图论 - Little Blue Book High School Vol.12: Graph Theory
|
44 |
+
|
45 |
+
* 小蓝书高中卷13 组合极值 - Little Blue Book High School Vol.13: Extremal Combinatorics
|
46 |
+
|
47 |
+
* 小蓝书高中卷14 高中数学竞赛中的解题方法与策略 - Little Blue Book High School Vol.14: Problem-Solving Methods & Strategies for Math Competitions
|
48 |
+
|
49 |
+
## Potential Usages
|
50 |
+
|
51 |
+
This dataset is a resource for AI researchers and developers, with key applications including:
|
52 |
+
|
53 |
+
* Training & Fine-Tuning – Enhancing large language models’ capabilities in advanced mathematical reasoning.
|
54 |
+
|
55 |
+
* AI Evaluation – Benchmarking the problem-solving proficiency and logical rigor of AI systems.
|
56 |
+
|
57 |
+
* Formal Verification – Formalizing problems into mathematical languages (e.g., LEAN) to evaluate AI's reasoning capability with formal methods.
|
58 |
+
|
59 |
+
* Comparative Analysis – Systematically assessing reasoning skills across different models and methodologies.
|
60 |
+
|
61 |
+
## How to Use
|
62 |
+
|
63 |
+
We provide the raw data (*.tex) and the processed dataset, including calculation, proof, text and images they referred to.
|
64 |
+
|
65 |
+
A case in `calculation`.
|
66 |
+
|
67 |
+
```json
|
68 |
+
{
|
69 |
+
"source_file": "./raw_volume-zh/volume1/chapter1.tex",
|
70 |
+
"problem_type": "calculation",
|
71 |
+
"problem": "例1. 设集合 $M=\\left\\{x |{\\frac{a x-5}{x^{2}-a}}<0,\\,x\\in\\mathbb{R}\\right\\}$ \n(1)当 $a=4$ 时,化简集合 $M$ ;\n(2)若 $3\\in M,$ ,且 $5\\notin M,$ 求实数a的取值范围.",
|
72 |
+
"solution": "分析: 化简集合 $M$, 实际上就是解不等式 ${\\frac{a x-5}{x^{2}-a}}<0.$ \n解: (1) 当 $a=4$ 时,有\n$$\n{\\frac{4x-5}{x^{2}-4}}<0\\,, \n$$\n即\n$$\n\\left(x-\\frac{5}{4}\\right)(x+2)(x-2)<0. \n$$\n$x<-2$ 或 ${\\frac{5}{4}}<x<2.$ \n所以 $M=(-\\infty,-2)\\cup\\bigl({\\frac{5}{4}}, 2\\bigr).$ \n(2)由 $3\\in M,$ 得 ${\\frac{3a-5}{3^{2}-a}}<0$,即 $\\left(a-\\frac{5}{3}\\right)(a-9)\\geqslant0$ ,所以\n$$\na<{\\frac{5}{3}}或a>9. \n$$\n由 $5\\notin M$ 得, ${\\frac{5a-5}{5^{2}-a}}\\geqslant0$ 或 $5^{2}-a=0$ ,所以\n$$\n1\\leq a\\leq25. \n$$\n可得 $x\\in\\left[1,{\\frac{5}{3}}\\right)\\cup\\left(9,25\\right]$.\n说明: $5\\notin M$ 隐含了条件 $5^{2}-a=$ 0,这是容易被忽视的.\n由概括原则我们知道,判断一个对象 $x$ 是否为集合 $S$ 的元素,等价于判断 $x$ 是否具有性质 $P$.",
|
73 |
+
"remark": "",
|
74 |
+
"figures": []
|
75 |
+
}
|
76 |
+
```
|
77 |
+
|
78 |
+
`source_file`: Path to the original .tex source file containing this problem.
|
79 |
+
|
80 |
+
`problem_type`: Problem category (e.g., "calculation", "proof", etc.).
|
81 |
+
|
82 |
+
`problem`: Complete problem statement in LaTeX format, including sub-questions.
|
83 |
+
|
84 |
+
`solution`: Step-by-step solution with mathematical derivations in LaTeX.
|
85 |
+
|
86 |
+
`remark`: Additional notes or comments about the problem (empty if none).
|
87 |
+
|
88 |
+
`figures`: Array containing any associated diagram files (empty if none).
|
89 |
+
|
90 |
+
|
91 |
+
## Citation
|
92 |
+
|
93 |
+
|
94 |
+
If you use the BlueMO dataset in your research, please consider citing it as follows:
|
95 |
+
|
96 |
+
```bibtex
|
97 |
+
@misc{bluemo2024,
|
98 |
+
title={BlueMO: A comprehensive collection of challenging mathematical olympiad problems from the little blue book series},
|
99 |
+
author={Chen, Yizhou and Luo, Yifan and Zhang, Yifan and Yuan, Yang},
|
100 |
+
year={2024},
|
101 |
+
}
|
102 |
+
```
|
processed_dataset/calculation/0001.json
ADDED
@@ -0,0 +1,8 @@
|
|
|
|
|
|
|
|
|
|
|
|
|
|
|
|
|
|
|
1 |
+
{
|
2 |
+
"source_file": "./raw_volume-zh/volume1/chapter1.tex",
|
3 |
+
"problem_type": "calculation",
|
4 |
+
"problem": "例1. 设集合 $M=\\left\\{x |{\\frac{a x-5}{x^{2}-a}}<0,\\,x\\in\\mathbb{R}\\right\\}$ \n(1)当 $a=4$ 时,化简集合 $M$ ;\n(2)若 $3\\in M,$ ,且 $5\\notin M,$ 求实数a的取值范围.",
|
5 |
+
"solution": "分析: 化简集合 $M$, 实际上就是解不等式 ${\\frac{a x-5}{x^{2}-a}}<0.$ \n解: (1) 当 $a=4$ 时,有\n$$\n{\\frac{4x-5}{x^{2}-4}}<0\\,, \n$$\n即\n$$\n\\left(x-\\frac{5}{4}\\right)(x+2)(x-2)<0. \n$$\n$x<-2$ 或 ${\\frac{5}{4}}<x<2.$ \n所以 $M=(-\\infty,-2)\\cup\\bigl({\\frac{5}{4}}, 2\\bigr).$ \n(2)由 $3\\in M,$ 得 ${\\frac{3a-5}{3^{2}-a}}<0$,即 $\\left(a-\\frac{5}{3}\\right)(a-9)\\geqslant0$ ,所以\n$$\na<{\\frac{5}{3}}或a>9. \n$$\n由 $5\\notin M$ 得, ${\\frac{5a-5}{5^{2}-a}}\\geqslant0$ 或 $5^{2}-a=0$ ,所以\n$$\n1\\leq a\\leq25. \n$$\n可得 $x\\in\\left[1,{\\frac{5}{3}}\\right)\\cup\\left(9,25\\right]$.\n说明: $5\\notin M$ 隐含了条件 $5^{2}-a=$ 0,这是容易被忽视的.\n由概括原则我们知道,判断一个对象 $x$ 是否为集合 $S$ 的元素,等价于判断 $x$ 是否具有性质 $P$.",
|
6 |
+
"remark": "",
|
7 |
+
"figures": []
|
8 |
+
}
|
processed_dataset/calculation/0002.json
ADDED
@@ -0,0 +1,8 @@
|
|
|
|
|
|
|
|
|
|
|
|
|
|
|
|
|
|
|
1 |
+
{
|
2 |
+
"source_file": "./raw_volume-zh/volume1/chapter1.tex",
|
3 |
+
"problem_type": "calculation",
|
4 |
+
"problem": "例4. 设关于 $x$ 的不等式 $\\left|x-{\\frac{(a+1)^{2}}{2}}\\right|\\leq{\\frac{(a-1)^{2}}{2}}$ 和 $x^{2}-3(a+1)x+2(3a+1)\\leq0\\ (a\\in\\mathbb{R})$ 的解集依次为 $A$、$B$,求使 $A\\,\\subseteq\\,B$ 的实数a 的取值范围.",
|
5 |
+
"solution": "分析: 要由 $A\\subseteq B$ 求出a的范围,必须先求出$A$和 $B$.\n解: 由 $\\left|x-{\\frac{(a+1)^{2}}{2}}\\right|\\leqslant{\\frac{(a-1)^{2}}{2}}$, 得\n$$\n-\\frac{(a-1)^{2}}{2}\\leq x-\\frac{(a+1)^{2}}{2}\\leq\\frac{(a-1)^{2}}{2}, \n$$\n解之,得 $2a\\leq x\\leq a^{2}+1.$ 所以 $,A=\\{x\\mid2a\\leq x\\leq a^{2}+1\\}$ \n由 $x^{2}-3(a+1)x+2(3a+1)\\leq0$,得\n$$\n(x-2)[x-(3a+1)]\\leq0. \n$$\n当 $a\\geq{\\frac{1}{3}}$ 时, $B = \\{ x \\mid 2 \\leq x \\leq 3a+1 \\}$ ;当 $a<{\\frac{1}{3}}$时,$B=\\{x \\mid 3a+1 \\leq x \\leq 2 \\}.$ \n因为 $A\\subseteq B$, 所以\n$$\n\\begin{align*}\n\\left\\{\n\\begin{aligned}\n a \\geq \\frac{1}{3}, \\\\\n 2a \\geq 2,\\\\\n a^2+1 \\leq 3a+1,\n\\end{aligned}\n\\right.\n\\end{align*}\n$$ \n或\n$$\n\\begin{align*}\n\\left\\{\n\\begin{aligned}\n a < \\frac{1}{3}, \\\\\n 2a \\geq 3a+1,\\\\\n a^2+1 \\leq 2.\n\\end{aligned}\n\\right.\n\\end{align*}\n$$ \n解之,得 $1\\leq a\\leq3$ 或 $a=-1.$ \n所以,a 的取值范围是[1, 3]U ${\\{-1}\\}.$ \n说明: 上述解答是通过对参数 $a$ 的分类讨论完成的,其实还有更直接的解法.\n方程的角度看 $A\\subseteq B$ 等价于方程 $x^{2}-3(a+1)x+2(3a+1)=0$ 在区间$(-\\infty,2a]$ 和 $[a^{2}+1,\\ +\\infty)$ 内各有一个实根.\n $f(x)\\,=\\,x^{2} - 3(a+1)x+2(3a+1)$ ,由 $A\\subseteq B$, 得\n$$\n\\begin{align*}\n\\left\\{\n\\begin{aligned}\n f(2a) \\leq 0, \\\\\n f(a^2+1) \\leq 0,\\\\\n\\end{aligned}\n\\right.\n\\end{align*}\n\\longrightarrow 1\\leq a \\leq 3 \\text{或} a=-1.$$",
|
6 |
+
"remark": "",
|
7 |
+
"figures": []
|
8 |
+
}
|
processed_dataset/calculation/0003.json
ADDED
@@ -0,0 +1,8 @@
|
|
|
|
|
|
|
|
|
|
|
|
|
|
|
|
|
|
|
1 |
+
{
|
2 |
+
"source_file": "./raw_volume-zh/volume1/chapter1.tex",
|
3 |
+
"problem_type": "calculation",
|
4 |
+
"problem": "例5. 设实数$a<b$, $D=[a\\,,\\,b]$ ,函数 $f(x)=k-\\sqrt{x+2}\\,,\\;x\\in D$ 的值域为 $E$. 若 $D=E$, 求实数 $k$ 的取值范围.",
|
5 |
+
"solution": "解: 易知, 当 $x \\geqslant-2$ 时 $f(x)=k-\\sqrt{x+2}$ 为减函数.\n所以 $D=E=[a, b]$ 等价于方程组\n$$\n\\left\\{\\begin{array}{l}\nk-\\sqrt{a+2}=b, \\\\\nk-\\sqrt{b+2}=a\n\\end{array}\\right.\n$$\n有实数解, 且 $a<b$.\n(1)一(2)得\n$$\n\\begin{aligned}\n& \\sqrt{b+2}-\\sqrt{a+2}=b-a, \\\\\n& \\frac{b-a}{\\sqrt{b+2}+\\sqrt{a+2}}=b-a .\n\\end{aligned}\n$$\n因为 $a<b$, 所以\n$$\n\\sqrt{b+2}+\\sqrt{a+2}=1,\n$$\n即\n$$\n\\sqrt{a+2}=1-\\sqrt{b+2} \\text {. }\n$$\n代入式(1)得\n$$\nk=b+1-\\sqrt{b+2} .\n$$\n令 $\\sqrt{b+2}=t$. 由式 (3) 知 $0 \\leqslant t \\leqslant 1$. 于是, 有\n$$\nk=t^2-t-1=\\left(t-\\frac{1}{2}\\right)^2-\\frac{5}{4} .\n$$\n故所求 $k$ 的范围是 $-\\frac{5}{4} \\leqslant k \\leqslant-1$.\n如果 $A 、 B$ 是两个相等的数集, 那么可以得到 $A=B$ 的两个非常有用的必要条件:\n(1) 两个集合的元素之和相等;\n(2) 两个集合的元素之积相等.",
|
6 |
+
"remark": "",
|
7 |
+
"figures": []
|
8 |
+
}
|
processed_dataset/calculation/0004.json
ADDED
@@ -0,0 +1,8 @@
|
|
|
|
|
|
|
|
|
|
|
|
|
|
|
|
|
|
|
1 |
+
{
|
2 |
+
"source_file": "./raw_volume-zh/volume1/chapter2.tex",
|
3 |
+
"problem_type": "calculation",
|
4 |
+
"problem": "例1. 已知 $A=\\left\\{x \\mid x^2+x-6=0\\right\\}, B=\\{x \\mid m x+1=0\\}$, 且 $A \\cup B=A$, 求实数 $m$ 的取值范围.",
|
5 |
+
"solution": "分析: 关键是如何理解和运用 $A \\cup B=A$ 这个条件.\n注意到 $A \\cup B=A \\Leftrightarrow B \\subseteq A$, 用列举法表示 $A$, 即可写出 $B$ 的各种情形, 但不要忘了 $B=\\varnothing$ 的情形!\n解: $A=\\left\\{x \\mid x^2+x-6=0\\right\\}=\\{-3,2\\} . B=\\{x \\mid m x+1=0\\}$ 至多有一个元素.\n因为 $A \\cup B=A$, 所以 $B \\subseteq A$. 因此, $B=\\{-3\\}$, 或 $B=\\{2\\}$, 或 $B=\\varnothing$, 即\n$$\n-3 m+1=0 \\text {, 或 } 2 m+1=0 \\text {, 或 } m=0 \\text {, }\n$$\n解得 $m=\\frac{1}{3}$ 或 $-\\frac{1}{2}$ 或 0 .\n故实数 $m \\in\\left\\{\\frac{1}{3},-\\frac{1}{2}, 0\\right\\}$.",
|
6 |
+
"remark": "",
|
7 |
+
"figures": []
|
8 |
+
}
|
processed_dataset/calculation/0005.json
ADDED
@@ -0,0 +1,8 @@
|
|
|
|
|
|
|
|
|
|
|
|
|
|
|
|
|
|
|
1 |
+
{
|
2 |
+
"source_file": "./raw_volume-zh/volume1/chapter2.tex",
|
3 |
+
"problem_type": "calculation",
|
4 |
+
"problem": "例2. 已知集合 $A=\\left\\{x \\mid x^2-2 x-3 \\leqslant 0\\right\\}, B=\\left\\{x \\mid x^2+p x+q<0\\right\\}$, 若 $A \\cap B=\\{x \\mid-1 \\leqslant x \\leqslant 2\\}$. 求 $p 、 q$ 的取值范围.",
|
5 |
+
"solution": "解:\n$$\nA=\\left\\{x \\mid x^2-2 x-3 \\leqslant 0\\right\\}=[-1,3] .\n$$\n设 $x^2+p x+q=0$ 的两根为 $x_1 、 x_2, x_1<x_2$. 则\n$$\n\\begin{gathered}\nx^2+p x+q=\\left(x-x_1\\right)\\left(x-x_2\\right), \\\\\nB=\\left(x_1, x_2\\right) .\n\\end{gathered}\n$$\n由 $A \\cap B=[-1,3] \\cap\\left(x_1, x_2\\right)=[-1,2)$, 得\n$$\n\\left\\{\\begin{array}{l}\nx_1<-1, \\\\\nx_2=2 .\n\\end{array}\\right.\n$$\n由韦达定理, 得\n$$\n\\begin{gathered}\nx_1+x_2=x_1+2=-p, \\\\\nx_1 x_2=2 x_1=q .\n\\end{gathered}\n$$\n因为 $x_1<-1$, 所以\n$$\n\\begin{gathered}\n-p-2<-1, \\\\\n\\frac{q}{2}<-1 .\n\\end{gathered}\n$$\n故所求 $p 、 q$ 的范围分别是 $p>-1 、 q<-2$.",
|
6 |
+
"remark": "",
|
7 |
+
"figures": []
|
8 |
+
}
|
processed_dataset/calculation/0006.json
ADDED
@@ -0,0 +1,10 @@
|
|
|
|
|
|
|
|
|
|
|
|
|
|
|
|
|
|
|
|
|
|
|
1 |
+
{
|
2 |
+
"source_file": "./raw_volume-zh/volume1/chapter2.tex",
|
3 |
+
"problem_type": "calculation",
|
4 |
+
"problem": "例3. 设 $A 、 B$ 都是不超过 9 的正整数组成的全集 $U$ 的子集, $A \\cap B= \\{2\\},\\left(\\complement_U A\\right) \\cap\\left(\\complement_U B\\right)=\\{1,9\\},\\left(\\complement_U A\\right) \\cap B=\\{4,6,8\\}$, 求 $A \\backslash B$.",
|
5 |
+
"solution": "分析:直接进行集合间的运算和推理似乎较难人手, 但我们可从维恩图(<FilePath:./images/volume1/figures/fig-c2e3.png>)中得到解题思路的提示.\n解因为 $\\complement_U(A \\cup B)=\\left(\\complement_U A\\right) \\cap\\left(\\complement_U B\\right)= \\{1,9\\}$, 所以\n$$\nA \\cup B=\\{2,3,4,5,6,7,8\\} .\n$$\n又\n$$\n\\begin{gathered}\nA \\cap B=\\{2\\}, \\\\\n\\left(\\complement_U A\\right) \\cap B=\\{4,6,8\\}, \\\\\nB=U \\cap B=\\left(A \\cup \\complement_U A\\right) \\cap B \\\\\n=(A \\cap B) \\cup\\left(\\left(\\complement_U A\\right) \\cap B\\right) \\\\\n=\\{2,4,6,8\\} .\n\\end{gathered}\n$$\n所以\n$$\n\\begin{aligned}\nB & =U \\cap B=\\left(A \\cup \\complement_U A\\right) \\cap B \\\\\n& =(A \\cap B) \\cup\\left(\\left(\\complement_U A\\right) \\cap B\\right) \\\\\n& =\\{2,4,6,8\\} .\n\\end{aligned}\n$$\n所以, $A \\backslash B=(A \\cup B) \\backslash B=\\{3,5,7\\}$.",
|
6 |
+
"remark": "",
|
7 |
+
"figures": [
|
8 |
+
"./images/volume1/figures/fig-c2e3.png"
|
9 |
+
]
|
10 |
+
}
|
processed_dataset/calculation/0007.json
ADDED
@@ -0,0 +1,8 @@
|
|
|
|
|
|
|
|
|
|
|
|
|
|
|
|
|
|
|
1 |
+
{
|
2 |
+
"source_file": "./raw_volume-zh/volume1/chapter2.tex",
|
3 |
+
"problem_type": "calculation",
|
4 |
+
"problem": "例4. 已知集合 $A=\\{(x, y) \\mid a x+y=1\\}, B=\\{(x, y) \\mid x+a y=1\\}$, $C=\\left\\{(x, y) \\mid x^2+y^2=1\\right\\}$. 问:\n(1) 当 $a$ 取何值时, $(A \\cup B) \\cap C$ 为含有两个元素的集合?\n(2) 当 $a$ 取何值时, $(A \\cup B) \\cap C$ 为含有三个元素的集合?",
|
5 |
+
"solution": "分析:因为 $(A \\cup B) \\cap C=(A \\cap C) \\cup(B \\cap C)$, 故可从解 $A \\cap C$ 及 $B \\cap C$ 对应的方程组人手.\n解: $(A \\cup B) \\cap C=(A \\cap C) \\cup(B \\cap C), A \\cap C$ 与 $B \\cap C$ 分别为方程组\n(i) $\\left\\{\\begin{array}{l}a x+y=1, \\\\ x^2+y^2=1,\\end{array}\\right.$\n(ii) $\\left\\{\\begin{array}{l}x+a y=1, \\\\ x^2+y^2=1\\end{array}\\right.$\n的解集.\n由 (i) 解得 $(x, y)=(0,1),\\left(\\frac{2 a}{1+a^2}, \\frac{1-a^2}{1+a^2}\\right)$;\n由(ii) 解得 $(x, y)=(1,0),\\left(\\frac{1-a^2}{1+a^2}, \\frac{2 a}{1+a^2}\\right)$.\n(1) 使 $(A \\cup B) \\cap C$ 恰有两个元素的情况只有两种可能:\n<1> $\\left\\{\\begin{array}{l}\\frac{2 a}{1+a^2}=0, \\\\ \\frac{1-a^2}{1+a^2}=1 ;\\end{array}\\right.$\n<2> $\\left\\{\\begin{array}{l}\\frac{2 a}{1+a^2}=1, \\\\ \\frac{1-a^2}{1+a^2}=0 .\\end{array}\\right.$\n由<1>得 $a=0$; 由<2>得 $a=1$.\n故当 $a=0$ 或 1 时, $(A \\cup B) \\cap C$ 恰有两个元素.\n(2) 使 $(A \\cup B) \\cap C$ 恰有三个元素的情况是\n$$\n\\frac{2 a}{1+a^2}=\\frac{1-a^2}{1+a^2}\n$$\n解得 $a=-1 \\pm \\sqrt{2}$.\n故当 $a=-1 \\pm \\sqrt{2}$ 时, $(A \\cup B) \\cap C$ 恰有三个元素.",
|
6 |
+
"remark": "",
|
7 |
+
"figures": []
|
8 |
+
}
|
processed_dataset/calculation/0008.json
ADDED
@@ -0,0 +1,8 @@
|
|
|
|
|
|
|
|
|
|
|
|
|
|
|
|
|
|
|
1 |
+
{
|
2 |
+
"source_file": "./raw_volume-zh/volume1/chapter2.tex",
|
3 |
+
"problem_type": "calculation",
|
4 |
+
"problem": "例5. 已知集合 $A=\\left\\{(x, y) \\mid \\frac{y-3}{x-2}=a+1\\right\\}, B=\\{(x, y) \\mid(a^2- 1) x+(a-1) y=15\\}$, 且 $A \\cap B=\\varnothing$, 求 $a$ 的值.",
|
5 |
+
"solution": "分析:当 $a=1$ 时, $B=\\varnothing$, 这时 $A \\cap B=\\varnothing$; 当 $a \\neq 1$ 时, $A \\cap B=\\varnothing$, 即 $A 、 B$ 对应的直线无公共点.\n解由 $\\frac{y-3}{x-2}=a+1$, 得\n$$\n(a+1) x-y-2 a+1=0, \\text { 且 } x \\neq 2 .\n$$\n这表明集合 $A$ 表示一条缺一个点的直线.\n而\n$$\n\\left(a^2-1\\right) x+(a-1) y=15,\n$$\n当 $a \\neq 1$ 时,表示一条直线; 当 $a=1$ 时,满足等式的点 $(x, y)$ 不存在.\n因此,当且仅当以下三种情况之一发生时, $A \\cap B=\\varnothing$.\n(1)当 $a=1$ 时, $B=\\varnothing$, 显然有 $A \\cap B=\\varnothing$.\n(2)当 $a=-1$ 时, $A$ 表示直线 $y=3(x \\neq 2), B$ 表示直线 $y=-\\frac{15}{2}$, 它们互相平行.\n所以, $A \\cap B=\\varnothing$.\n(3)当 $a \\neq \\pm 1$ 时, 直线 (1) 与 (2) 相交.\n但直线 (1) 上缺一点 $(2,3)$, 令 $(2, 3) \\in B$, 得\n$$\n\\left(a^2-1\\right) \\cdot 2+(a-1) \\cdot 3=15,\n$$\n解得 $a=-4$ 或 $a=\\frac{5}{2}$.\n综上所述, $a \\in\\left\\{-4,-1,1, \\frac{5}{2}\\right\\}$.\n说明 $a \\neq 1$ 时, $A \\cap B=\\varnothing$, 并不表明直线 (1) 与 (2) 必须平行, 由于直线 (1) 上缺了一个点 $(2,3)$, 当直线 (2) 穿过点 $(2,3)$ 时, 同样有 $A \\cap B=\\varnothing$.",
|
6 |
+
"remark": "",
|
7 |
+
"figures": []
|
8 |
+
}
|
processed_dataset/calculation/0009.json
ADDED
@@ -0,0 +1,10 @@
|
|
|
|
|
|
|
|
|
|
|
|
|
|
|
|
|
|
|
|
|
|
|
1 |
+
{
|
2 |
+
"source_file": "./raw_volume-zh/volume1/chapter2.tex",
|
3 |
+
"problem_type": "calculation",
|
4 |
+
"problem": "例7. 已知集合 $A 、 B 、 C$ (不必相异)的并集\n$$\nA \\cup B \\cup C=\\{1,2, \\cdots, 2005\\},\n$$\n求满足条件的有序三元组 $(A, B, C)$ 的个数.",
|
5 |
+
"solution": "解: 由图(<FilePath:./images/volume1/figures/fig-c2e7.png>)可知,表示集合 $A 、 B 、 C$ 的 3 个圆交出了 7 个区域.\n这表明,在求 $A \\cup B \\cup C$ 时, 1 , $2, \\cdots, 2005$ 中每一个数都有 7 种选择.\n所以,所求的有序三元组的个数为 $7^{2005}$.",
|
6 |
+
"remark": "",
|
7 |
+
"figures": [
|
8 |
+
"./images/volume1/figures/fig-c2e7.png"
|
9 |
+
]
|
10 |
+
}
|
processed_dataset/calculation/0010.json
ADDED
@@ -0,0 +1,8 @@
|
|
|
|
|
|
|
|
|
|
|
|
|
|
|
|
|
|
|
1 |
+
{
|
2 |
+
"source_file": "./raw_volume-zh/volume1/chapter3.tex",
|
3 |
+
"problem_type": "calculation",
|
4 |
+
"problem": "例1. 设集合 $A=\\left\\{(x, y, z) \\mid \\log _{\\frac{1}{4}}\\left(x^4+y^4+z^4+1\\right) \\geqslant \\log _4 \\frac{1}{x}+\\log _4 \\frac{1}{y}+\\log _4 \\frac{1}{z}-1\\right\\}$. 求 $|A|$.",
|
5 |
+
"solution": "分析:无疑应从考察 $(x, y, z)$ 满足的条件人手.\n解由 $\\log _{\\frac{1}{4}}\\left(x^4+y^4+z^4+1\\right) \\geqslant \\log _4 \\frac{1}{x}+\\log _4 \\frac{1}{y}+\\log _4 \\frac{1}{z}-1$ 得\n$$\nx^4+y^4+z^4+1 \\leqslant 4 x y z, x, y, z>0 .\n$$\n又由算术几何平均不等式, 得\n$$\nx^4+y^4+z^4+1 \\geqslant 4 x y z,\n$$\n其中等号当且仅当 $x=y=z=1$ 时成立.\n于是\n$$\n\\begin{gathered}\nx^4+y^4+z^4+1=4 x y z . \\\\\nx=y=z=1 .\n\\end{gathered}\n$$\n从而所以, $|A|=1$.",
|
6 |
+
"remark": "",
|
7 |
+
"figures": []
|
8 |
+
}
|
processed_dataset/calculation/0011.json
ADDED
@@ -0,0 +1,8 @@
|
|
|
|
|
|
|
|
|
|
|
|
|
|
|
|
|
|
|
1 |
+
{
|
2 |
+
"source_file": "./raw_volume-zh/volume1/chapter3.tex",
|
3 |
+
"problem_type": "calculation",
|
4 |
+
"problem": "例2. 设集合 $A=\\{a \\mid 1 \\leqslant a \\leqslant 2000, a=4 k+1, k \\in \\mathbf{Z}\\}$, 集合 $B= \\{b \\mid 1 \\leqslant b \\leqslant 3000, b=3 k-1, k \\in \\mathbf{Z}\\}$. 求 $|A \\cap B|$.",
|
5 |
+
"solution": "分析:令 $4 k+1=3 m-1$, 得 $m=\\frac{4 k+2}{3}=k+1+\\frac{k-1}{3}$. 因 $m \\in \\mathbf{Z}$, 所以 $3 \\mid k-1$. 令 $k-1=3 r, r \\in \\mathbf{Z}$, 得 $m=4 r+2$. 这时 $b=12 r+5$, 故 $A \\cap B$的元素是形如 $12 r+5$ 的整数.\n解形如 $4 k+1$ 的数可分为 3 类:\n$$\n12 l+1,12 l+5,12 l+9(l \\in \\mathbf{Z}),\n$$\n其中只有形如 $12 l+5$ 的数是形如 $3 k-1$ 的数.\n令\n$$\n1 \\leqslant 12 l+5 \\leqslant 2000(l \\in \\mathbf{Z}),\n$$\n得 $0 \\leqslant l \\leqslant 166$. 所以, $A \\cap B=\\{5,17, \\cdots, 1997\\}$.\n所以, $|A \\cap B|=167$.\n以上两例, 我们都是采用列举出集合的全部元素的办法来求其元素的数目.\n对于一些较为复杂的集合, 这种方法是很难奏效的, 这时必须另辟蹊径.",
|
6 |
+
"remark": "",
|
7 |
+
"figures": []
|
8 |
+
}
|
processed_dataset/calculation/0012.json
ADDED
@@ -0,0 +1,8 @@
|
|
|
|
|
|
|
|
|
|
|
|
|
|
|
|
|
|
|
1 |
+
{
|
2 |
+
"source_file": "./raw_volume-zh/volume1/chapter3.tex",
|
3 |
+
"problem_type": "calculation",
|
4 |
+
"problem": "例3. 设 $\\left(a_1, a_2, \\cdots, a_n\\right)$ 是集合 $\\{1,2, \\cdots, n\\}$ 中 $n$ 个元素的一个排列, 记所有满足\n$$\nk \\mid 2\\left(a_1+a_2+\\cdots+a_k\\right), k=1,2, \\cdots, n\n$$\n的排列 $\\left(a_1, a_2, \\cdots, a_n\\right)$ 的集合为 $A_n$. 求 $\\left|A_n\\right|$ 的值.",
|
5 |
+
"solution": "分析:显然 $1\\left|2 a_1, n\\right| 2\\left(a_1+a_2+\\cdots+a_n\\right)$, 我们需要研究当 $2 \\leqslant k \\leqslant n-1$ 时, $k \\mid 2\\left(a_1+a_2+\\cdots+a_k\\right)$ 应满足的条件.\n对于一般的 $k$, 我们没有更好的办法来表示 $a_1+a_2+\\cdots+a_k$, 但当 $k=n-1$ 时, 显然有 $a_1+a_2+\\cdots+ a_{n-1}=1+2+\\cdots+n-a_n {=} \\frac{n(n+1)}{2}-a_n$, 于是 $n-1 \\mid 2\\left(a_1+a_2+\\cdots+a_{n-1}\\right)$ 等价于 $n-1 \\mid n(n+1)-2 a_n$, 问题转化为对 $a_n$ 的研究.\n解设 $F_n=\\left|A_n\\right|$. 容易算出 $F_1=1, F_2=2, F_3=6$.\n当 $n>3$ 时,对于任意 $\\left(a_1, a_2, \\cdots, a_n\\right) \\in A_n$, 有\n$$\n\\begin{aligned}\n& 2\\left(a_1+a_2+\\cdots+a_{n-1}\\right) \\\\\n= & n(n+1)-2 a_n \\equiv 2-2 a_n(\\bmod (n-1)) .\n\\end{aligned}\n$$\n由 $A_n$ 的定义, 必有\n$$\nn-1 \\mid 2-2 a_n \\text {. }\n$$\n故 $a_n=1$,或 $a_n=n$, 或 $a_n=\\frac{n+1}{2}$.\n(1) 若 $a_n=\\frac{n+1}{2}$, 则\n$$\n\\begin{aligned}\n2\\left(a_1+a_2+\\cdots+a_{n-2}\\right) & =n(n+1)-2 a_{n-1}-(n+1) \\\\\n& =n^2-1-2 a_{n-1} \\\\\n& \\equiv 3-2 a_{n-1}(\\bmod (n-2)) .\n\\end{aligned}\n$$\n从而有\n$$\nn-2 \\mid 3-2 a_{n-1} .\n$$\n解得 $a_{n-1}=\\frac{n+1}{2}$. 于是 $a_{n-1}=a_n$, 矛盾.\n(2) 若 $a_n=n$, 则 $\\left(a_1, a_2, \\cdots, a_{n-1}, n\\right)$ 与 $A_{n-1}$ 的元素 $\\left(a_1, a_2, \\cdots, a_{n-1}\\right)$ 形成一一对应关系.\n所以, 这样的排列共有 $F_{n-1}$ 种.\n(3) 若 $a_n=1$, 则 $\\left(a_1-1, a_2-1, \\cdots, a_{n-1}-1\\right)$ 是集合 $\\{1,2, \\cdots, n-1\\}$ 中 $n-1$ 个元素的一个排列.\n由\n$$\n\\begin{aligned}\n& 2\\left[\\left(a_1-1\\right)+\\left(a_2-1\\right)+\\cdots+\\left(a_k-1\\right)\\right] \\\\\n= & 2\\left(a_1+a_2+\\cdots+a_k\\right)-2 k \\\\\n\\equiv & 0(\\bmod k) \\\\\n\\Leftrightarrow & \\left(a_1-1, a_2-1, \\cdots, a_{n-1}-1\\right) \\in A_{n-1}\n\\end{aligned}\n$$\n知 $\\left(a_1, a_2, \\cdots, a_{n-1}, 1\\right)$ 与 $A_{n-1}$ 的元素 $\\left(a_1-1, a_2-1, \\cdots, a_{n-1}-1\\right)$ 之间也形成一一对应关系.\n故这样的排列也有 $F_{n-1}$ 种.\n由(2)、(3), 可建立递推关系\n$$\nF_n=2 F_{n-1}, n>3 .\n$$\n由 $F_3=6$, 得 $F_n=3 \\cdot 2^{n-2}(n \\geqslant 3)$.\n综上, 当 $n=1$ 时, $F_1=1$; 当 $n=2$ 时, $F_2=2$; 当 $n \\geqslant 3$ 时, $F_n=3 \\cdot 2^{n-2}$.\n说明这里, 我们通过建立 $F_n$ 与 $F_{n-1}$ 之间的联系 (递推关系) 来达到求解的目的。",
|
6 |
+
"remark": "",
|
7 |
+
"figures": []
|
8 |
+
}
|
processed_dataset/calculation/0013.json
ADDED
@@ -0,0 +1,8 @@
|
|
|
|
|
|
|
|
|
|
|
|
|
|
|
|
|
|
|
1 |
+
{
|
2 |
+
"source_file": "./raw_volume-zh/volume1/chapter3.tex",
|
3 |
+
"problem_type": "calculation",
|
4 |
+
"problem": "例8. 设 $S$ 是一个由正整数组成的集合, 具有如下性质: 对任意 $x \\in S$, 在 $S$ 中去掉 $x$ 后, 剩下的数的算术平均值都是正整数, 并且 $1 \\in S, 2002$ 是 $S$ 中的最大元.\n求 $|S|$ 的最大值.",
|
5 |
+
"solution": "分析:显然 1 是 $S$ 中的最小元.\n设 $S$ 的元素为 $1=x_1<x_2<\\cdots<x_n=$ 2002 , 由 $\\frac{\\sum_{i=1}^n x_i-x_j}{n-1} \\in \\mathbf{N}^*$, 我们来估计 $|S|$ 的范围.\n解设 $S$ 中的元素为\n$$\n1=x_1<x_2<\\cdots<x_n=2002,\n$$\n则对于 $1 \\leqslant j \\leqslant n$, 均有\n$$\ny_j=\\frac{\\left(\\sum_{i=1}^n x_i\\right)-x_j}{n-1} \\in \\mathbf{N}^* .\n$$\n从而, 对任意 $1 \\leqslant i<j \\leqslant n$, 都有\n$$\ny_i-y_j=\\frac{x_j-x_i}{n-1} \\in \\mathbf{N}^* .\n$$\n特别地, 应有 $n-1 \\mid(2002-1)$, 即\n$$\nn-1 \\mid 2001 .\n$$\n另一方面, 对于 $1<j \\leqslant n$, 均有\n$$\n\\begin{gathered}\nx_j-1=\\left(y_1-y_j\\right)(n-1), \\\\\nn-1 \\mid\\left(x_j-1\\right) . \\\\\n\\left(x_j-1\\right)-\\left(x_{j-1}-1\\right), \\text { 所以 } \\\\\n(n-1) \\mid\\left(x_j-x_{j-1}\\right)(j=2, \\cdots, n),\n\\end{gathered}\n$$\n从而\n$$\n\\begin{aligned}\n\\text { 又 } x_j-x_{j-1}= & \\left(x_j-1\\right)-\\left(x_{j-1}-1\\right), \\text { 所以 } \\\\\n& (n-1) \\mid\\left(x_j-x_{j-1}\\right)(j=2, \\cdots, n),\n\\end{aligned}\n$$\n于是\n$$\n\\begin{aligned}\nx_n-1 & =\\left(x_n-x_{n-1}\\right)+\\left(x_{n-1}-x_{n-2}\\right)+\\cdots+\\left(x_2-1\\right) \\\\\n& \\geqslant(n-1)+(n-1)+\\cdots+(n-1)=(n-1)^2,\n\\end{aligned}\n$$\n即 $(n-1)^2 \\leqslant 2001, n \\leqslant 45$. 结合 $n-1 \\mid 2001$, 知 $n=2,4,24,30$, 故 $n \\leqslant 30$.\n另一方面, 令 $x_j=29 j-28,1 \\leqslant j \\leqslant 29, x_{30}=2002$, 则 $S=\\left\\{x_1, x_2, \\cdots\\right.$, $x_{30}$ 具有题述性质.\n所以, $|S|$ 的最大值为 30 .\n说明先估计 $|S|=n$ 的上界, 即 $n \\leqslant 30$, 再构造一个实例说明 $n=30$ 是可以达到的, 从而知 $n$ 的最大值为 30 . 这种“先估计, 再构造”的方法在解决离散型最值问题时经常被用到.",
|
6 |
+
"remark": "",
|
7 |
+
"figures": []
|
8 |
+
}
|
processed_dataset/calculation/0014.json
ADDED
@@ -0,0 +1,8 @@
|
|
|
|
|
|
|
|
|
|
|
|
|
|
|
|
|
|
|
1 |
+
{
|
2 |
+
"source_file": "./raw_volume-zh/volume1/chapter3.tex",
|
3 |
+
"problem_type": "calculation",
|
4 |
+
"problem": "例9. 试求出同时满足下列条件的集合 $S$ 的元素个数的最大值:\n(1) $S$ 中的每个元素都是不超过 100 的正整数;\n(2) 对于 $S$ 中的任意两个不同的元素 $a 、 b$, 都存在 $S$ 中的另外一个元素 $c$, 使得 $a+b$ 与 $c$ 的最大公约数等于 1 ;\n(3) 对于 $S$ 中的任意两个不同的元素 $a 、 b$, 都存在 $S$ 中的另外一个元素 $c$, 使得 $a+b$ 与 $c$ 的最大公约数大于 1 .",
|
5 |
+
"solution": "分析:若 $a+b$ 为质数,则条件 (3) 无法满足.\n而 101 就是一个质数,这说明数组 $\\{1,100\\},\\{2,99\\}, \\cdots,\\{50,51\\}$ 中, 每组的两个数不同时在 $S$ 中.\n那么在每组数中各取一个数组成的集合是否满足所有条件呢?\n解构造 50 个数组:\n$$\n\\{1,100\\},\\{2,99\\}, \\cdots,\\{50,51\\},\n$$\n每个数组中的两个数之和是 101 .\n由于 101 是质数, 在 $S$ 中不存在元素 $c$, 使得 101 与 $c$ 的最大公约数大于 1. 因此, 在 $S$ 中不可能同时含有上述数组中的同一数组中的两个数.\n由抽屉原理可知,集合 $S$ 中元素的个数不大于 50 .\n另一方面, 我们构造集合 $A=\\{2,1,3,5,7, \\cdots, 95,97\\}$. 此集合含有 2 和小于 98 的 49 个奇数.\n下面说明集合 $A$ 满足题设条件.\n对于集合 $A$ 中的任意两个元素 $a$ 和 $b$ :\n(i) 若 $a=2$, 则 $b$ 是奇数.\n若 $b=1$, 易见 $A$ 中存在元素 $c$ 满足题设条件;\n若 $3 \\leqslant b \\leqslant 95$, 则 $A$ 中元素 1 与 $a+b$ 的最大公约数等于 $1, A$ 中元素 $b+2$ 与 $a+b$ 的最大公约数是 $b+2$ 大于 1 ;\n若 $b=97$, 易见 $A$ 中存在元素 $c$ 满足题设条件.\n(ii) 若 $a 、 b$ 都不等于 2 , 则 $a 、 b$ 都是奇数, $a+b$ 是偶数.\n于是, $a+b$ 与 2 的最大公约数是 2 大于 1 , 且 $a+b$ 与 $1 、 89 、 91$ 中的某个数必互质.\n所以,集合 $A$ 满足题设条件.\n因此,集合 $S$ 的元素个数的最大值是 50 .",
|
6 |
+
"remark": "",
|
7 |
+
"figures": []
|
8 |
+
}
|
processed_dataset/calculation/0015.json
ADDED
@@ -0,0 +1,8 @@
|
|
|
|
|
|
|
|
|
|
|
|
|
|
|
|
|
|
|
1 |
+
{
|
2 |
+
"source_file": "./raw_volume-zh/volume1/chapter4.tex",
|
3 |
+
"problem_type": "calculation",
|
4 |
+
"problem": "例1. 试将集合 $\\{1,2, \\cdots, 1989\\}$ 分为 117 个互不相交的子集 $A_i ( i=1, 2, \\cdots, 117)$, 使得:\n(1) 每个 $A_i$ 都含有 17 个元素;\n(2) 所有 $A_i$ 中各元素之和都相同.",
|
5 |
+
"solution": "分析:因为 $1989=117 \\times 17$, 故可将 $\\{1,2, \\cdots, 1989\\}$ 顺次分成 17 段, 每段含 117 个数.\n显然, 只要把每段的 117 个数适当地分别放人 $A_1, A_2, \\cdots, A_{117}$ 中以使条件 (2)满足,问题就解决了.\n解将集合 $\\{1,2, \\cdots, 1989\\}$ 中的数从小到大顺次分成 17 段,每段含 117 个数.\n从第 4 段数开始, 将偶数段的数从小到大依次放人 $A_1, A_2, \\cdots, A_{117}$ 中, 并将奇数段的数从大到小依次放人这 117 个子集中.\n易见,所有集合中的 14个数之和都相等.\n于是问题归结为如何将前三段数 $\\{1,2, \\cdots, 351\\}$ 每 3 个一组分别放人每个集中, 且使每组 3 数之和都相等.\n把这些数中 3 的倍数抽出来从大到小排好: $\\{351,348,345, \\cdots, 6,3\\}$ , 共 117 个数,依次放人 $A_1, A_2, \\cdots, A_{117}$ 中.\n其余的 234 个数从小到大排列并分成两段, 每段 117 个数, 即 $\\{1,2,4,5,7, \\cdots, 173 , 175\\}$ 和 $\\{176,178,179, \\cdots, 349,350\\}$. 将这两段数分别顺次放人 $A_1, A_2, \\cdots, A_{117}$ 之中便满足要求.\n事实上, 若将这两段数中的数顺次相加, 则其和为 $\\{177,180,183,186, \\cdots, 522,525\\}$. 由此可见, 放人每个 $A_i$ 的 3 数之和都是 528 .\n说明上述解法是通过具体地构造 $A_i(i=1,2, \\cdots, 117)$ 完成的.\n由此不难看出, 这种构造方式不是惟一的, 有兴趣者不妨一试.",
|
6 |
+
"remark": "",
|
7 |
+
"figures": []
|
8 |
+
}
|
processed_dataset/calculation/0016.json
ADDED
@@ -0,0 +1,8 @@
|
|
|
|
|
|
|
|
|
|
|
|
|
|
|
|
|
|
|
1 |
+
{
|
2 |
+
"source_file": "./raw_volume-zh/volume1/chapter4.tex",
|
3 |
+
"problem_type": "calculation",
|
4 |
+
"problem": "例3. 对一个由非负整数组成的集合 $S$, 定义 $r_s(n)$ 为满足下述条件的有序对 $\\left(s_1, s_2\\right)$ 的对数:\n$$\ns_1 \\in S, s_2 \\in S, s_1 \\neq s_2, \\text { 且 } s_1+s_2=n .\n$$\n问: 是否能将非负整数集分划为两个集合 $A$ 和 $B$, 使得对任意 $n$, 均有 $r_A(n)=r_B(n)$ ?",
|
5 |
+
"solution": "分析:整数有多种表示形式, 其中二进制表示的每位数字只有 0 和 1 这两种选择.\n由于是将 $S$ 分划为两个集合 $A 、 B$, 对每个固定的 $n$, 满足 $s_1+s_2=n$ 的非负整数对 $\\left(s_1, s_2\\right)$ 是有限的, 用二进制数来讨论 $\\left(s_1, s_2\\right)$ 在 $A$ 和 $B$ 中的分配情况似乎较有利.\n解存在上述的分划.\n将所有二进制表示下数码 1 出现偶数个的非负整数归人集合 $A$, 其余的非负整数归人 $B$, 则 $A 、 B$ 是非负整数集 $N$ 的分划.\n注意到, 对 $A$ 中满足 $a_1+a_2=n, a_1 \\neq a_2, a_1, a_2 \\in A$ 的数对 $\\left(a_1, a_2\\right)$, 由于 $a_1 \\neq a_2$, 因此在二进制表示下 $a_1$ 与 $a_2$ 必有一位上的数码不同, 从右到左看, 第 1 个不同数码的数位上, 改变 $a_1 、 a_2$ 在该位上的数码, 分别得到 $b_1 、 b_2$, 则 $b_1 、 b_2 \\in B$, 且 $b_1 \\neq b_2, b_1+b_2=n$. 这个将 $\\left(a_1, a_2\\right)$ 对应到 $\\left(b_1, b_2\\right)$ 的映射是一一对应, 因此 $r_A(n)=r_B(n)$.\n说明这是一个存在性问题.\n我们是利用二进制数构造 $S$ 的 $2-$ 分划 $A$ 、 $B$, 然后通过建立 $A$ 中有序对集 $\\{\\left(a_1, a_2\\right) \\mid a_1, a_2 \\in A, a_1 \\neq a_2, a_1+a_2=n\\}$ 与 $B$ 中有序对集 $\\left\\{\\left(b_1, b_2\\right) \\mid b_1, b_2 \\in B, b_1 \\neq b_2, b_1+b_2=n\\right\\}$ 的一一对应来解决的.\n利用一一对应解决计数问题的方法就是所谓的配对原理.",
|
6 |
+
"remark": "",
|
7 |
+
"figures": []
|
8 |
+
}
|
processed_dataset/calculation/0017.json
ADDED
@@ -0,0 +1,8 @@
|
|
|
|
|
|
|
|
|
|
|
|
|
|
|
|
|
|
|
1 |
+
{
|
2 |
+
"source_file": "./raw_volume-zh/volume1/chapter4.tex",
|
3 |
+
"problem_type": "calculation",
|
4 |
+
"problem": "例4. 设集合 $A=\\{1,2, \\cdots, m\\}$. 求最小的正整数 $m$, 使得对 $A$ 的任意一个 14 -分划 $A_1, A_2, \\cdots, A_{14}$, 一定存在某个集合 $A_i(1 \\leqslant i \\leqslant 14)$, 在 $A_i$ 中有两个元素 $a 、 b$, 满足 $b<a \\leqslant \\frac{4}{3} b$.",
|
5 |
+
"solution": "分析:由于要考虑的是一种极端情况, 我们来作一张元素、集合从属关系的表: 从 1 开始, 由小到大每 14 个数为一组, 依次填人表中的每一列中 (如表 4-1). 填满 4 列后, 观察发现: 去掉右下角的数 56 后, 子集 $A_1, A_2, \\cdots$, $A_{13}$ 中每一个都有 4 个元素, 而 $A_{14}$ 有 3 个元素, 这时 $A_1, A_2, \\cdots, A_{14}$ 任何一个中都不存在两个元素满足题中的不等式.\n故 $m \\geqslant 56$.\n表 4-1\n$\\begin{array}{llllll}A_1 & 1 & 15 & 29 & 43 & \\cdots\\end{array}$\n$\\begin{array}{llllll}A_2 & 2 & 16 & 30 & 44 & \\cdots\\end{array}$\n$\\begin{array}{llllll}A_3 & 3 & 17 & 31 & 45 & \\cdots\\end{array}$\n$\\begin{array}{cccccc}\\vdots & \\vdots & \\vdots & \\vdots & \\vdots & \\vdots \\\\ A_{12} & 12 & 26 & 40 & 54 & \\cdots \\\\ A_{13} & 13 & 27 & 41 & 55 & \\cdots \\\\ A_{14} & 14 & 28 & 42 & 56 & \\cdots\\end{array}$\n表 4-2\n$\\begin{array}{lllll}A_1 & 1 & 15 & 29 & 43\\end{array}$\n$\\begin{array}{lllll}A_2 & 2 & 16 & 30 & 44\\end{array}$\n$\\begin{array}{lllll}A_3 & 3 & 17 & 31 & 45\\end{array}$\n$\\begin{array}{ccccc}\\vdots & \\vdots & \\vdots & \\vdots & \\vdots \\\\ A_{12} & 12 & 26 & 40 & 54 \\\\ A_{13} & 13 & 27 & 41 & 55 \\\\ A_{14} & 14 & 28 & 42 & \\end{array}$\n解如表 4-2, 第 $i$ 行的数即为子集 $A_i$ 的元素.\n这时 $\\left|A_i\\right|=4(i=1,2, \\cdots, 13),\\left|A_{14}\\right|=3$. 显然, 14 个子集每一个都不存在两个元素满足题中不等式.\n所以, $m \\geqslant 56$.\n另一方面, 若 $m=56$, 则对 $A$ 的任意分划 $A_1, A_2, \\cdots, A_{14}$, 数 $42 , 43, \\cdots, 56$ 中, 必有两个数属于同一个 $A_i$, 取此二数为 $a 、 b$, 则\n$$\n42 \\leqslant a<b \\leqslant 56=\\frac{4}{3} \\cdot 42 \\leqslant \\frac{4}{3} a .\n$$\n综上所述, 所求 $m$ 的最小正整数值为 56 .\n另解若 $m<56$, 令 $A_i=\\{a \\mid a \\equiv i(\\bmod 14), a \\in A\\}$, 则对任意 $a, b \\in A_i(i=1,2, \\cdots, 14), b<a$, 均有 $56>a>b$, 且 $a-b \\geqslant 14$. 故 $b<a- 14<42$. 于是\n$$\n\\frac{a}{b}=1+\\frac{a-b}{b} \\geqslant 1+\\frac{14}{b}>1+\\frac{14}{42}=\\frac{4}{3} .\n$$\n所以, $m \\geqslant 56$.\n后同前解.",
|
6 |
+
"remark": "",
|
7 |
+
"figures": []
|
8 |
+
}
|
processed_dataset/calculation/0018.json
ADDED
@@ -0,0 +1,8 @@
|
|
|
|
|
|
|
|
|
|
|
|
|
|
|
|
|
|
|
1 |
+
{
|
2 |
+
"source_file": "./raw_volume-zh/volume1/chapter4.tex",
|
3 |
+
"problem_type": "calculation",
|
4 |
+
"problem": "例10. 设 $A=\\{1,2, \\cdots, 2002\\}, M=\\{1001,2003,3005\\}$. 对 $A$ 的任一非空子集 $B$, 当 $B$ 中任意两数之和不属于 $M$ 时, 称 $B$ 为 $M$-自由集.\n如果 $A= A_1 \\cup A_2, A_1 \\cap A_2=\\varnothing$, 且 $A_1 、 A_2$ 均为 $M$-自由集, 那么称有序对 $\\left(A_1, A_2\\right)$ 为 $A$ 的一个 $M$-划分.\n试求 $A$ 的所有 $M$-划分的个数.",
|
5 |
+
"solution": "解:对 $m, n \\in A$, 若 $m+n=1001$ 或 2003 或 3005 , 则称 $m$ 与 $n$ “有关”.\n易知, 与 1 有关的数仅有 1000 和 2002 ,与 1000 和 2002 有关的都是 1 和 1003 , 与 1003 有关的为 1000 和 2002 .\n将 $1,1003,1000,2002$ 分为两组 $\\{1,1003\\},\\{1000,2002\\}$, 其中一组中的数仅与另一组中的数有关, 我们将这样的两组叫做一个 “组对”. 同样可划分其他各组对 $\\{2,1004\\},\\{999,2001\\} ;\\{3,1005\\},\\{998,2000\\} ; \\cdots ;\\{500,1502\\},\\{501,1503\\} ;\\{1001\\},\\{1002\\}$.\n这样 $A$ 中的 2002 个数被分划成 501 个组对,共 1002 组.\n由于任意数与且只与对应另一组有关, 所以, 若一组对中一组在 $A_1$ 中, 另一组必在 $A_2$ 中.\n反之亦然, 且 $A_1$ 与 $A_2$ 中不再有有关的数.\n故 $A$ 的 $M$ 一划分的个数为 $2^{501}$.",
|
6 |
+
"remark": "",
|
7 |
+
"figures": []
|
8 |
+
}
|
processed_dataset/calculation/0019.json
ADDED
@@ -0,0 +1,8 @@
|
|
|
|
|
|
|
|
|
|
|
|
|
|
|
|
|
|
|
1 |
+
{
|
2 |
+
"source_file": "./raw_volume-zh/volume1/chapter5.tex",
|
3 |
+
"problem_type": "calculation",
|
4 |
+
"problem": "例4. 已知集合 $A=\\{1,2, \\cdots, 10\\}$. 求集合 $A$ 的具有下列性质的子集个数: 每个子集至少含有 2 个元素, 且每个子集中任何两个元素的差的绝对值大于 1 .",
|
5 |
+
"solution": "分析:集合 $A$ 有 $2^{10}-1$ 个非空子集,逐一考察的工作只有交给计算机.\n像例 1 一样, 我们先来看看比 $A$ 的元素少一些的集合的情形.\n记集合 $A_i$ 符合条件的子集族为 $A_i^*,\\left|A_i^*\\right|=a_i$.\n$$\n\\begin{aligned}\nA_1 & =\\{1\\}, A_1^*=\\varnothing, a_1=0 ; \\\\\nA_2 & =\\{1,2\\}, A_2^*=\\varnothing, a_2=0 ; \\\\\nA_3 & =\\{1,2,3\\}, A_3^*=\\{\\{1,3\\}\\}, a_3=1 ; \\\\\nA_4 & =\\{1,2,3,4\\}, A_4^*=\\{\\{1,3\\},\\{1,4\\},\\{2,4\\}\\}, a_4=3 ; \\\\\nA_5 & =\\{1,2,3,4,5\\}, A_5^*=\\{\\{1,3\\},\\{1,4\\},\\{2,4\\},\\{1,3,5\\}, \\{1,5\\},\\{2,5\\},\\{3,5\\}\\}, a_5=7 . &\n\\end{aligned}\n$$\n我们来考察写出 $A_5^*$ 的过程, 这可以分作两步: 第一步写出 $A_4^*$ 的全部元素, 它们都不含元素 5 ; 第二步写出含 5 的子集, 它们是在 $A_3^*$ 的元素中添 5 所成, 或者是含 5 的二元子集, 即 $a_5=a_4+a_3+3$. 其实对 $A_4^* 、 A_3^*$ 有类似的结论: $a_4=a_3+a_2+2, a_3=a_2+a_1+1$. 我们可以将这个作法推广到一般。\n解设 $a_n$ 是集合 $\\{1,2, \\cdots, n\\}$ 的具有题设性质的子集个数.\n对于集合 $\\{1,2, \\cdots, n, n+1, n+2\\}$, 具有题设性质的子集可分为两类: 第一类子集不包含 $n+2$, 它们是集合 $\\{1,2, \\cdots, n, n+1\\}$ 的全部具有题设性质的子集, 共有 $a_{n-1}$ 个; 第二类子集包含 $n+2$, 它们是集合 $\\{1,2, \\cdots, n\\}$ 的每个具有题设性质的子集与 $\\{n+2\\}$ 的并集, 以及二元子集 $\\{1, n+2\\},\\{2$, $n+2\\}, \\cdots,\\{n, n+2\\}$, 共有 $a_n+n$ 个.\n于是, 我们有\n$$\na_{n+2}=a_{n+1}+a_n+n .\n$$\n易知, $a_1=a_2=0$, 因此 $a_3=1, a_4=3, a_5=7, a_6=14, a_7=26$, $a_8=46, a_9=79, a_{10}=133$.\n所以,所求子集的个数为 133 .\n说明上述解法的特点是将问题一般化,一般问题解决了, 特殊问题当然就解决了.\n这里用到了递推方法, 递推也是解决组合问题的常用方法之一.\n与上例相反,我们来看一个已知子集族求恰好包含这些子集的集合的阶的问题.",
|
6 |
+
"remark": "",
|
7 |
+
"figures": []
|
8 |
+
}
|
processed_dataset/calculation/0020.json
ADDED
@@ -0,0 +1,8 @@
|
|
|
|
|
|
|
|
|
|
|
|
|
|
|
|
|
|
|
1 |
+
{
|
2 |
+
"source_file": "./raw_volume-zh/volume1/chapter5.tex",
|
3 |
+
"problem_type": "calculation",
|
4 |
+
"problem": "例8. 设 $A \\subseteq\\{0,1,2, \\cdots, 29\\}$ 满足:对任何整数 $k$ 及 $A$ 中任意数 $a 、 b$ ( $a 、 b$ 可以相同), $a+b+30 k$ 均不是两个相邻整数之积.\n试定出所含元素个数最多的 $A$.",
|
5 |
+
"solution": "分析:因为当 $b=a$ 时, $2 a+30 k$ 均不是两个相邻整数之积, 故我们只需考察 $2 a$ 被 30 除的余数.\n解所求 $A$ 为 $\\{3 l+2 \\mid 0 \\leqslant l \\leqslant 9\\}$.\n设 $A$ 满足题中条件且 $|A|$ 最大.\n因为两个相邻整数之积被 30 除, 余数为 $0,2,6,12,20,26$. 则对任一 $a \\in A$, 有 $2 a \\neq 0,2,6,12,20,26(\\bmod 30)$ ,\n即 $a \\neq 0,1,3,6,10,13,15,16,18,21,25,28$, 因此, $A \\subseteq\\{2,4,5,7,8,9,11,12,14,17,19,20,22,23,24,26,27,29\\}$, 后一集合可分拆成下列 10 个子集的并, 其中每一个子集至多有一个元素包含在 $A$ 中: $\\{2,4\\}, \\{5,7\\},\\{8,12\\},\\{9,11\\},\\{14,22\\},\\{17,19\\},\\{20\\},\\{23,27\\},\\{24,26\\},\\{29\\}$, 故 $|A| \\leqslant 10$.\n若 $|A|=10$, 则每个子集恰好有一个元素包含在 $A$ 中, 因此, $20 \\in A, 29 \\in A$.\n由 $20 \\in A$ 知 $12 \\notin A$, 从而 $8 \\in A$, 这样 $4 \\notin A, 22 \\notin A, 24 \\notin A$. 因此 $2 \\in A, 14 \\in A, 26 \\in A$.\n由 $29 \\in A$ 知 $7 \\notin A, 27 \\notin A$, 从而 $5 \\in A, 23 \\in A$, 这样 $9 \\notin A, 19 \\notin A$, 因此 $11 \\in A, 17 \\in A$.\n综上所述,有 $A=\\{2,5,8,11,14,17,20,23,26,29\\}$, 此集合 $A$ 确实满足要求.",
|
6 |
+
"remark": "",
|
7 |
+
"figures": []
|
8 |
+
}
|
processed_dataset/calculation/0021.json
ADDED
@@ -0,0 +1,8 @@
|
|
|
|
|
|
|
|
|
|
|
|
|
|
|
|
|
|
|
1 |
+
{
|
2 |
+
"source_file": "./raw_volume-zh/volume1/chapter7.tex",
|
3 |
+
"problem_type": "calculation",
|
4 |
+
"problem": "例1. 已知 $y=\\frac{x^2}{10}-\\frac{x}{10}+\\frac{9}{5}$, 且 $y \\leqslant|x|$, 求 $x$ 的取值范围.",
|
5 |
+
"solution": "分析:为了去掉 $y \\leqslant|x|$ 中 $|x|$ 的绝对值符号, 自然要对 $x$ 进行分类: 当 $x \\geqslant 0$ 时, $y \\leqslant x$; 当 $x<0$ 时, $y \\leqslant-x$. 由此知, 本题应分两种情况讨论.\n解当 $x \\geqslant 0$ 时, 有 $y \\leqslant x$. 即亦即\n$$\n\\begin{aligned}\n& \\frac{x^2}{10}-\\frac{x}{10}+\\frac{9}{5} \\leqslant x, \\\\\n& (x-2)(x-9) \\leqslant 0 .\n\\end{aligned}\n$$\n解得 $2 \\leqslant x \\leqslant 9$.\n当 $x<0$ 时, 有 $y \\leqslant-x$. 即\n$$\n\\begin{aligned}\n& \\frac{x^2}{10}-\\frac{x}{10}+\\frac{9}{5} \\leqslant-x, \\\\\n& (x+3)(x+6) \\leqslant 0 .\n\\end{aligned}\n$$\n解得 $-6 \\leqslant x \\leqslant-3$.\n综上,所求 $x$ 的取值范围为 $[-6,-3] \\cup[2,9]$.\n说明以上解答是以绝对值的定义为标准进行分类的.\n注意不要漏掉了 $x=0$ 的情形,这里我们是将 $x=0$ 与 $x>0$ 并在一起考虑的,但并非任何时候都可以这么做!",
|
6 |
+
"remark": "",
|
7 |
+
"figures": []
|
8 |
+
}
|
processed_dataset/calculation/0022.json
ADDED
@@ -0,0 +1,8 @@
|
|
|
|
|
|
|
|
|
|
|
|
|
|
|
|
|
|
|
1 |
+
{
|
2 |
+
"source_file": "./raw_volume-zh/volume1/chapter7.tex",
|
3 |
+
"problem_type": "calculation",
|
4 |
+
"problem": "例2. 已知 $a>0, a \\neq 1$, 解关于 $x$ 的不等式:\n$$\n2 \\log _a(x-1)>\\log _a[1+a(x-2)] .\n$$",
|
5 |
+
"solution": "分析:解对数不等式必然要考虑对数函数的单调性.\n于是, 将底数 $a$ 分为 $0<a<1$ 和 $a>1$ 两种情形讨论.\n解 (1) 当 $0<a<1$ 时, 原不等式等价于\n$$\n\\begin{aligned}\n& \\left\\{\\begin{array}{l}\nx-1>0, \\\\\n1+a(x-2)>0, \\\\\n(x-1)^2<1+a x-2 a,\n\\end{array}\\right. \\\\\n& \\left\\{\\begin{array}{l}\nx>1, \\\\\nx>2-\\frac{1}{a}, \\\\\na<x<2 .\n\\end{array}\\right.\n\\end{aligned}\n$$\n即因为 $0<a<1$, 所以 $1>2-\\frac{1}{a}$. 所以此时原不等式的解为 $1<x<2$.\n(2) 当 $a>1$ 时,原不等式等价于\n$$\n\\begin{aligned}\n& \\left\\{\\begin{array}{l}\nx-1>0, \\\\\n1+a(x-2)>0, \\\\\n(x-1)^2>1+a x-2 a,\n\\end{array}\\right. \\\\\n& \\left\\{\\begin{array}{l}\nx>2-\\frac{1}{a}, \\\\\n(x-2)(x-a)>0 .\n\\end{array}\\right.\n\\end{aligned}\n$$\ni) 当 $1<a<2$ 时, 由(2)得 $x<a$ 或 $x>2$.\n因为 $a>1$, 所以 $a-\\left(2-\\frac{1}{a}\\right)=a+\\frac{1}{a}-2>2 \\sqrt{a \\cdot \\frac{1}{a}}-2=0$, 所以 $a>2-\\frac{1}{a}$.\n所以, 此时原不等式的解为 $2-\\frac{1}{a}<x<a$ 或 $x>2$.\nii) 当 $a \\geqslant 2$ 时,由 (2) 得 $x<2$ 或 $x>a$.\n因为 $a \\geqslant 2$, 所以 $2>2-\\frac{1}{a}$.\n所以, 此时原不等式的解为 $2-\\frac{1}{a}<x<2$ 或 $x>a$.\n综上, 当 $0<a<1$ 时, 原不等式的解集为 $(1,2)$; 当 $1<a<2$ 时, 原不等式的解集为 $\\left(2-\\frac{1}{a}, a\\right) \\cup(2,+\\infty)$; 当 $a \\geqslant 2$ 时, 原不等式的解集为 $\\left(2-\\frac{1}{a}, 2\\right) \\cup(a,+\\infty)$.\n说明上述解答中的分类讨论有如下特点:\n1. 讨论是围绕参数 $a$ 展开的.\n2. 采用了二级分类的方式: 第一级的分类是由对数函数的单调性引起的, 我们将参数 $a$ 分为两大类: (1) $0<a<1$, (2) $a>1$; 第二级的分类是为了比较不等式 (2) 对应的方程 $(x-2)(x-a)=0$ 的两根的大小, 我们将 $a>1$ 又分成两小类: i) $1<a<2$, ii) $a \\geqslant 2$. 每级分类都严格遵循分类原则, 这种分类方式可推”到更多级的情形.\n3. 最后的结论是依不同情况下解的状况重新按一级分类叙述的.",
|
6 |
+
"remark": "",
|
7 |
+
"figures": []
|
8 |
+
}
|
processed_dataset/calculation/0023.json
ADDED
@@ -0,0 +1,8 @@
|
|
|
|
|
|
|
|
|
|
|
|
|
|
|
|
|
|
|
1 |
+
{
|
2 |
+
"source_file": "./raw_volume-zh/volume1/chapter7.tex",
|
3 |
+
"problem_type": "calculation",
|
4 |
+
"problem": "例3. 设 $n$ 是一个正整数.\n安先写出 $n$ 个不同的正整数, 然后艾夫删除了其中的某些数 (可以不删, 但不能全删), 同时在每个剩下的数的前面放上 “+”号或“-”号, 再对这些数求和.\n如果计算结果能被 2003 整除, 则艾夫获胜,否则安获胜.\n问谁有必胜策略?",
|
5 |
+
"solution": "分析:$n$ 个不同整数所成的集合 $M$ 有 $2^n-1$ 个不同的非空子集.\n当 $2^n- 1>2003$ 时, 必有两个不同的子集的元素和关于模 2003 同余.\n设这两个子集为 $A 、 B$, 且 $A \\cap B=C$. 则集合 $A \\backslash C$ 与 $B \\backslash C$ 的元素和关于模 2003 仍同余.\n这时, 艾夫只要在集合 $A \\backslash C$ 的元素前加“十” 号,在 $B \\backslash C$ 的元素前加“一”号, 而将其他元素全删除, 即可获胜.\n取 $n \\geqslant 11$, 便有 $2^n-1>2003$.\n那么, 当 $n \\leqslant 10$ 时有什么结果呢? 这时只要安写下整数 $1,2, \\cdots, 2^{n-1} (n \\leqslant 10)$ 中的若干个, 则已立于不败之地.\n因为艾夫无论怎么做, 所得的和都只能在一 1023 与 1023 之间,且不等于 0 .\n解当 $n \\leqslant 10$ 时, 安有必胜策略.\n为此, 他可写出整数 $1,2, \\cdots, 2^{n-1}$. 因为 $1+2+\\cdots+2^{n-1}=2^n-1 \\leqslant 2^{10}-1=1023$, 所以, 艾夫可能得到的和只能在一 1023 与 1023 之间.\n由二进制数的表示的惟一性及添加正、负号的办法知, 艾夫得到的和也不可能为 0 . 所以,艾夫必败无疑.\n当 $n \\geqslant 11$ 时, 艾夫有必胜的策略.\n设安写出的整数所成之集为 $M$. 因为 $2^n-1 \\geqslant 2^{11}-1>2003$, 所以 $M$ 的非空子集数大于 2003 . 因而, 一定存在 $M$ 的两个不同子集, 例如 $A$ 和 $B$, 使得 $A$ 中数的和与 $B$ 中数的和关于模 2003 同余.\n如果艾夫将“+”号放在集合 $A \\backslash B$ 中的数的前面, 将“-”号放在集合 $B \\backslash A$ 中的数的前面, 并删除 $M$ 中所有其余的数,则艾夫必胜.",
|
6 |
+
"remark": "",
|
7 |
+
"figures": []
|
8 |
+
}
|
processed_dataset/calculation/0024.json
ADDED
@@ -0,0 +1,8 @@
|
|
|
|
|
|
|
|
|
|
|
|
|
|
|
|
|
|
|
1 |
+
{
|
2 |
+
"source_file": "./raw_volume-zh/volume1/chapter7.tex",
|
3 |
+
"problem_type": "calculation",
|
4 |
+
"problem": "例4。 彼得有 25 名同班同学(他自己未计人数目 25 之内). 已知这 25 名同学在班内的朋友数目各不相同, 试问彼得在该班有多少名朋友?",
|
5 |
+
"solution": "分析:因为彼得也可能是同班同学的朋友, 所以彼得的 25 名同学的朋友数分别只能是 $0,1,2, \\cdots, 24,25$ 之一, 且互不相同.\n如果在彼得的同学中存在孤独者 (无朋友者), 则另 24 个同学的朋友数分别为 $1,2, \\cdots, 24$; 否则, 彼得的 25 个同学的朋友数分别为 $1,2, \\cdots, 25$. 看来我们得分如上两种情形讨论.\n解分两种情形讨论.\n第一种情形假定某位同学在班上的朋友数为 0 . 则除了这位孤独者和彼得以外,其他同学每人在班上的朋友数不多于 24 . 因为这些同学总共 24 人, 每人在班上的朋友数不同, 所以他们的朋友数依次为 $1,2, \\cdots, 24$.\n约定将朋友数为 $1,2, \\cdots, 12$ 的同学编为 $A$ 组, 将朋友数为 $13,14, \\cdots$, 24 的同学编为 $B$ 组.\n将各组同学的朋友数求和, 分别得到\n$$\n\\begin{aligned}\n& S(A)=1+2+\\cdots+12=78, \\\\\n& S(B)=13+14+\\cdots+24=222 .\n\\end{aligned}\n$$\n设 $A$ 组中有 $k$ 名同学是彼得的朋友.\n则 $A$ 组同学在 $B$ 组中的朋友数总和不多于 $S(A)-k$, 另外 $B$ 组同学在本组中的朋友数总和不超过 $12 \\times 11$. 因此, 彼得在 $B$ 组中的朋友数不少于\n$$\nS(B)-12 \\times 11-(S(A)-k)=12+k,\n$$\n但 $B$ 组总共只有 12 人, 所以只能是 $k=0 . A$ 组中没有彼得的朋友 $(A$ 组同学也没有在本组中的朋友), $B$ 组的每位同学都是彼得的朋友.\n对此情形, 彼得在班上有 12 名朋友.\n第二种情形设班上没有孤独者, 每个人都有朋友.\n朋友数各不相同, 最多可达 25 人.\n约定将朋友数为 $1,2, \\cdots, 12$ 的同学编人 $A$ 组; 将朋友数为 $13,14, \\cdots, 25$ 的同学编人 $B$ 组.\n将各组同学的朋友数求和, 分别得到\n$$\n\\begin{aligned}\n& S(A)=1+2+\\cdots+12=78, \\\\\n& S(B)=13+14+\\cdots+25=247 .\n\\end{aligned}\n$$\n设 $A$ 组中有 $k$ 名同学是彼得的朋友.\n则 $A$ 组同学在 $B$ 组中的朋友数总和不多于 $S(A)-k$. 另外, $B$ 组同学在本组中的朋友数总共不超过 $13 \\times 12$. 于是, 彼得在 $B$ 组中的朋友数不少于\n$$\nS(B)-13 \\times 12-(S(A)-k)=13+k,\n$$\n但 $B$ 组总共只有 13 人, 所以 $k=0 . A$ 组中无彼得的朋友, $B$ 组中的每位同学都是彼得的朋友.\n对此情形, 彼得在班上有 13 名朋友.",
|
6 |
+
"remark": "",
|
7 |
+
"figures": []
|
8 |
+
}
|
processed_dataset/calculation/0025.json
ADDED
@@ -0,0 +1,8 @@
|
|
|
|
|
|
|
|
|
|
|
|
|
|
|
|
|
|
|
1 |
+
{
|
2 |
+
"source_file": "./raw_volume-zh/volume1/chapter7.tex",
|
3 |
+
"problem_type": "calculation",
|
4 |
+
"problem": "例6. 对任意 $n, k \\in \\mathbf{N}^*$, 令 $S=1^n+2^n+3^n+\\cdots+k^n$. 求 $S$ 被 3 除所得的余数.",
|
5 |
+
"solution": "分析:因为 $(3 m)^n \\equiv 0(\\bmod 3),(3 m+1)^n \\equiv 1(\\bmod 3)$, $(3 m+2)^{2 r} \\equiv 1(\\bmod 3),(3 m+2)^{2 r+1} \\equiv 2(\\bmod 3)$, 所以对 $n$ 按奇偶性分类是自然的.\n解 (1) 当 $n$ 为奇数时, 不妨设 $n=2 l-1, l \\in \\mathbf{N}^*$. 对 $m \\in \\mathbf{N}^*$, 如果 $3 \\times m$, 则 $m^2 \\equiv 1(\\bmod 3) \\Rightarrow m^{2 l} \\equiv 1(\\bmod 3) \\Rightarrow m^{2 l-1} \\equiv m^{2(l-1)+1} \\equiv m(\\bmod 3)$; 如果 $3 \\mid m$, 则 $m^{2 l-1} \\equiv 0 \\equiv m(\\bmod 3)$. 于是, 当 $n$ 为奇数时, 对 $m \\in \\mathbf{N}$, 总有 $m^n \\equiv m(\\bmod 3)$. 从而\n$$\n\\begin{aligned}\nS & \\equiv 1+2+3+\\cdots+k \\\\\n& \\equiv(1+2+3)+(4+5+6)+\\cdots(\\bmod 3) .\n\\end{aligned}\n$$\n当 $k=3 t+3$ 或 $k=3 t+2$ 时, 就有\n$$\nS \\equiv 0(\\bmod 3)(t \\in \\mathbf{N})\n$$\n当 $k=3 t+1$ 时, 就有\n$$\n\\begin{aligned}\nS \\equiv & (1+2+3)+(4+5+6)+\\cdots \\\\\n& +[(k-3)+(k-2)+(k-1)]+k \\\\\n\\equiv & 1(\\bmod 3)(t \\in \\mathbf{N}) .\n\\end{aligned}\n$$\n(2) 当 $n$ 为偶数时, 对 $m \\in \\mathbf{N}$, 由 (1) 知, $3 \\times m \\Rightarrow m^n \\equiv 1(\\bmod 3), 3 \\mid m \\Rightarrow m^n \\equiv 0(\\bmod 3)$. 于是\n$$\nS \\equiv(1+1+0)+(1+1+0)+\\cdots(\\bmod 3) .\n$$\n当 $k=3 t+3(t \\in \\mathbf{N})$ 时, $(1+1+0)$ 共有 $t+1$ 组, 故 $S \\equiv(t+1)(1+1+0) \\equiv 2 t+2(\\bmod 3)$;\n当 $k=3 t+2(t \\in \\mathbf{N})$ 时, $(1+1+0)$ 共有 $t$ 组, 且 $(k-1)^n \\equiv k^n \\equiv 1(\\bmod 3)$, 故 $S \\equiv 2 t+1+1 \\equiv 2 t+2(\\bmod 3)$;\n当 $k=3 t+1(t \\in \\mathbf{N})$ 时, $(1+1+0)$ 共有 $t$ 组, 且 $k^n \\equiv 1(\\bmod 3)$, 故 $S \\equiv 2 t+1(\\bmod 3)$.\n综合 (1)、(2) 可知:\n当 $n$ 为奇正整数时,有当 $n$ 为偶正整数时,有\n$$\nS \\equiv\\left\\{\\begin{array}{l}\n0, k=9 t+4 \\text { 或 } 9 t+8 \\text { 或 } 9 t+9, \\\\\n1, k=9 t+1 \\text { 或 } 9 t+5 \\text { 或 } 9 t+6,(\\bmod 3)(t \\in \\mathbf{N}) . \\\\\n2, k=9 t+2 \\text { 或 } 9 t+3 \\text { 或 } 9 t+7\n\\end{array}\\right.\n$$\n说明这是一个两级分类的例子.\n首先是对 $n$ 按奇偶性(模 2 的剩余类) 分成两大类,然后又将每一大类对 $k$ 按模 3 的剩余类分成三个小类.",
|
6 |
+
"remark": "",
|
7 |
+
"figures": []
|
8 |
+
}
|
processed_dataset/calculation/0026.json
ADDED
@@ -0,0 +1,8 @@
|
|
|
|
|
|
|
|
|
|
|
|
|
|
|
|
|
|
|
1 |
+
{
|
2 |
+
"source_file": "./raw_volume-zh/volume1/chapter7.tex",
|
3 |
+
"problem_type": "calculation",
|
4 |
+
"problem": "例7. 求集合 $B 、 C$, 使得 $B \\cup C=\\{1,2, \\cdots, 10\\}$, 并且 $C$ 的元素乘积等于 $B$ 的元素和.",
|
5 |
+
"solution": "分析:这实际上是求特殊条件下集合方程的解.\n注意到集合 $B$ 的元素和 $\\leqslant 1+2+\\cdots+10=55$, 而 $1 \\cdot 2 \\cdot 3 \\cdot 4 \\cdot 5=120$, 故知集合 $C$ 至多有 4 个元素.\n这样我们可以按 $|C|$ 的可能值分 4 类来讨论.\n解因为 $1+2+\\cdots+10=55<120=1 \\cdot 2 \\cdot 3 \\cdot 4 \\cdot 5$, 所以集合 $C$ 至多有 4 个元素.\n下面对 $|C|$ 分 4 种情况讨论.\n(1) $C$ 由一个元素构成.\n因为 $C$ 的元素乘积不超过 $10, B$ 的元素和至少为 $55-10=45$. 故此情况不成立.\n(2) $C$ 由两个元素 $x 、 y$ 构成.\n设 $x<y$, 则有 $x y=55-x-y$, 即\n$$\n(x+1)(y+1)=56 .\n$$\n因为 $x+1<y+1 \\leqslant 11$, 解得 $x=6, y=7$. 故 $C=\\{6,7\\}, B=\\{1,2,3,4,5,8,9,10\\}$.\n(3) $C$ 由三个元素 $x<y<z$ 构成.\n由题设得\n$$\nx y z=55-x-y-z .\n$$\n当 $x=1$ 时,解得 $y=4, z=10$. 因此, $C=\\{1,4,10\\}, B=\\{2,3,5$, $6,7,8,9\\}$.\n当 $x=2$ 时, 有 $2 y z+y+z=53$, 即 $(2 y+1)(2 z+1)=107$ 为质数.\n无解.\n若 $x \\geqslant 3$, 显然有 $x y z \\geqslant 3 \\times 4 \\times 5=60>55-x-y-z$. 无解.\n(4) $C$ 由四个元素 $x<y<z<t$ 构成.\n必有 $x=1$, 否则 $x y z t \\geqslant 2 \\times 3 \\times 4 \\times 5=120>55$. 这时\n$$\ny z t=54-y-z-t, 2 \\leqslant y<z<t .\n$$\n如 (3), $y \\geqslant 3$ 时无解.\n故 $y=2,2 z t+z+t=52$, 即 $(2 z+1)(2 t+1)= 105$. 解得 $z=3, t=7$. 从而, $C=\\{1,2,3,7\\}, B=\\{4,5,6,8,9,10\\}$.\n综上知, $B 、 C$ 有 3 组解.\n说明这里的分类并不是一眼看出来的, 但在经过了对两个集合的元素和与元素积的估计后, 这个分类就是自然的了, 而且后面的解答过程或多或少与这个估计有关.",
|
6 |
+
"remark": "",
|
7 |
+
"figures": []
|
8 |
+
}
|
processed_dataset/calculation/0027.json
ADDED
@@ -0,0 +1,8 @@
|
|
|
|
|
|
|
|
|
|
|
|
|
|
|
|
|
|
|
1 |
+
{
|
2 |
+
"source_file": "./raw_volume-zh/volume1/chapter7.tex",
|
3 |
+
"problem_type": "calculation",
|
4 |
+
"problem": "例8. 对任意的 $a>0, b>0$, 求 $\\min \\left\\{\\max \\left\\{\\frac{1}{a}, \\frac{1}{b}, a^2+b^2\\right\\}\\right\\}$ 的值.",
|
5 |
+
"solution": "分析:为了求出 $\\max \\left\\{\\frac{1}{a}, \\frac{1}{b}, a^2+b^2\\right\\}$, 我们来比较 $\\frac{1}{a} 、 \\frac{1}{b} 、 a^2+b^2$ 的大小.\n令 $\\frac{1}{a}=\\frac{1}{b}=a^2+b^2$, 得 $a=b=\\sqrt[3]{\\frac{1}{2}}$. 如设 $a \\geqslant b>0$, 则 $a 、 b 、 \\sqrt[3]{\\frac{1}{2}}$ 有三种顺序关系: $a \\geqslant b \\geqslant \\sqrt[3]{\\frac{1}{2}}, \\sqrt[3]{\\frac{1}{2}} \\geqslant a \\geqslant b, a \\geqslant \\sqrt[3]{\\frac{1}{2}} \\geqslant b$. 我们就以此分类.\n解不失一般性, 不妨设 $a \\geqslant b>0$, 则 $0<\\frac{1}{a} \\leqslant \\frac{1}{b}$. 令 $\\frac{1}{a}=\\frac{1}{b}=a^2+b^2$, 则 $a=b=\\sqrt[3]{\\frac{1}{2}}$.\n(1) 若 $a \\geqslant b \\geqslant \\sqrt[3]{\\frac{1}{2}}$, 则\n$$\n\\frac{1}{a} \\leqslant \\frac{1}{b} \\leqslant \\sqrt[3]{2}, a^2+b^2 \\geqslant 2 b^2 \\geqslant \\sqrt[3]{2} .\n$$\n所以 $\\max \\left\\{\\frac{1}{a}, \\frac{1}{b}, a^2+b^2\\right\\}=a^2+b^2 \\geqslant \\sqrt[3]{2}$. 从而当且仅当 $a=b=\\sqrt[3]{\\frac{1}{2}}$ 时,\n$$\n\\min \\left\\{\\max \\left\\{\\frac{1}{a}, \\frac{1}{b}, a^2+b^2\\right\\}\\right\\}=\\min \\left\\{a^2+b^2\\right\\}=\\sqrt[3]{2} ;\n$$\n(2) 若 $\\sqrt[3]{\\frac{1}{2}} \\geqslant a \\geqslant b>0$, 则\n$$\n\\frac{1}{b} \\geqslant \\frac{1}{a} \\geqslant \\sqrt[3]{2}, a^2+b^2 \\leqslant 2 a^2 \\leqslant \\sqrt[3]{2}\n$$\n所以 $\\max \\left\\{\\frac{1}{a}, \\frac{1}{b}, a^2+b^2\\right\\}=\\frac{1}{b} \\geqslant \\sqrt[3]{2}$. 从而当且仅当 $a=b=\\sqrt[3]{\\frac{1}{2}}$ 时,\n$$\n\\min \\left\\{\\max \\left\\{\\frac{1}{a}, \\frac{1}{b}, a^2+b^2\\right\\}\\right\\}=\\min \\left\\{\\frac{1}{b}\\right\\}=\\sqrt[3]{2} .\n$$\n(3)若 $a \\geqslant \\sqrt[3]{\\frac{1}{2}} \\geqslant b>0$, 则 $\\frac{1}{b} \\geqslant \\sqrt[3]{2} \\geqslant \\frac{1}{a}>0$.\n此时若 $\\frac{1}{b} \\geqslant a^2+b^2$, 则\n$$\n\\max \\left\\{\\frac{1}{a}, \\frac{1}{b}, a^2+b^2\\right\\}=\\frac{1}{b} \\geqslant \\sqrt[3]{2}\n$$\n若 $\\frac{1}{b} \\leqslant a^2+b^2$, 则\n$$\n\\max \\left\\{\\frac{1}{a}, \\frac{1}{b}, a^2+b^2\\right\\}=a^2+b^2 \\geqslant \\frac{1}{b} \\geqslant \\sqrt[3]{2} ;\n$$\n所以当且仅当 $a=b=\\sqrt[3]{\\frac{1}{2}}$ 时,\n$$\n\\min \\left\\{\\max \\left\\{\\frac{1}{a}, \\frac{1}{b}, a^2+b^2\\right\\}\\right\\}=\\sqrt[3]{2}\n$$\n综上所述: 当且仅当 $a=b=\\sqrt[3]{\\frac{1}{2}}$ 时,\n$$\n\\min \\left\\{\\max \\left\\{\\frac{1}{a}, \\frac{1}{b}, a^2+b^2\\right\\}\\right\\}=\\sqrt[3]{2} .\n$$",
|
6 |
+
"remark": "",
|
7 |
+
"figures": []
|
8 |
+
}
|
processed_dataset/calculation/0028.json
ADDED
@@ -0,0 +1,8 @@
|
|
|
|
|
|
|
|
|
|
|
|
|
|
|
|
|
|
|
1 |
+
{
|
2 |
+
"source_file": "./raw_volume-zh/volume1/chapter9.tex",
|
3 |
+
"problem_type": "calculation",
|
4 |
+
"problem": "例1. 设正整数 $a 、 b 、 c$ 为三角形三边长, $a+b=n, n \\in \\mathbf{N}^*, 1 \\leqslant c \\leqslant n-1$. 求这样的三角形的个数.",
|
5 |
+
"solution": "分析:设 $\\triangle A B C$ 的角 $A 、 B 、 C$ 的对应边分别为 $a 、 b 、 c$. 例 1 就是要计算有限集 $M=\\left\\{\\triangle A B C \\mid a+b=n, a, b \\in \\mathbf{N}^*, 1 \\leqslant c \\leqslant n-1\\right\\}$ 的阶.\n也就是要计算同时满足 $a+b>c, b+c>a, c+a>b$ 的三元正整数组 $\\{a, b, c\\}$ 的个数.\n解不妨设 $b \\geqslant a$, 则 $1 \\leqslant a \\leqslant\\left[\\frac{n}{2}\\right]$. 满足题设条件的三角形可分为两类:\n第一类: $c$ 为最大边.\n令 $a=i$, 则 $b=n-i, n-i \\leqslant c \\leqslant n-1$. 这样的三角形有 $(n-1)-(n-i)+1=i$ 个.\n第二类: $c$ 不为最大边.\n则 $b>c, c+a>b$, 故 $b-a=n-2 i, n-2 i< c<n-i$. 因此 $n-2 i+1 \\leqslant c \\leqslant n-i-1$. 这样的三角形有 $(n-i-1)- (n-2 i+1)+1=i-1$ 个.\n由加法原理, 满足题设条件的三角形的个数为\n$$\nf(n)=\\sum_{i=1}^{\\left[\\frac{n}{2}\\right]}(i+i-1)=\\sum_{i=1}^{\\left[\\frac{n}{2}\\right]}(2 i-1)=\\left(\\left[\\frac{n}{2}\\right]\\right)^2 .\n$$",
|
6 |
+
"remark": "",
|
7 |
+
"figures": []
|
8 |
+
}
|
processed_dataset/calculation/0029.json
ADDED
@@ -0,0 +1,8 @@
|
|
|
|
|
|
|
|
|
|
|
|
|
|
|
|
|
|
|
1 |
+
{
|
2 |
+
"source_file": "./raw_volume-zh/volume1/chapter9.tex",
|
3 |
+
"problem_type": "calculation",
|
4 |
+
"problem": "例2. 集合 $S=\\{1,2, \\cdots, 1990\\}$, 考察 $S$ 的 31 元子集.\n如果子集中 31 个元素之和可被 5 整除,则称为是好的.\n试求 $S$ 的好子集的个数.",
|
5 |
+
"solution": "分析:直接计算好子集的个数是困难的.\n考察 $S$ 的全部 31 元子集, 将其按子集元素和模 5 的剩余类分成 5 类, 直觉告诉我们, 每一类子集的个数似乎是相同的.\n果真是这样的吗?\n解我们来考察 $S$ 的全部 31 元子集, 这样的子集共有 $\\mathrm{C}_{1990}^{31}$ 个, 它们构成集合\n$$\nM=\\left\\{\\left\\{a_1, a_2, \\cdots, a_{31}\\right\\} \\mid\\left\\{a_1, a_2, \\cdots, a_{31}\\right\\} \\subset S\\right\\} .\n$$\n设 $\\left\\{a_1, a_2, \\cdots, a_{31}\\right\\} \\in M$, 其元素和被 5 除的余数为 $k$, 即\n$$\n\\sum_{i=1}^{31} a_i=k(\\bmod 5) .\n$$\n$k$ 只有 5 个可能值: $0,1,2,3,4$. 我们将所有 $k$ 值相同的 $M$ 的元素 ( $S$ 的 31 元子集) 归为一类, 得到 $M$ 的 5 个子集 $A_0, A_1, A_2, A_3$ 和 $A_4$. 显然 $A_0, A_1, \\cdots, A_4$ 是 $M$ 的一个分划,其中 $A_0$ 的元素就是 $S$ 的好子集.\n由于 $31 \\equiv 1(\\bmod 5)$, 所以当 $\\left\\{a_1, a_2, \\cdots, a_{31}\\right\\} \\in A_0$ 时, 即当 $\\sum_{i=1}^{31} a_i \\equiv 0(\\bmod 5)$ 时, 就有\n$$\n\\sum_{i=1}^{31}\\left(a_i+k\\right) \\equiv k(\\bmod 5) .\n$$\n故知 $\\left\\{a_1+k, a_2+k, \\cdots, a_{31}+k\\right\\} \\in A_k, k=1,2,3,4$, 这里当 $a_i+k>1990$ 时,将 $a_i+k$ 理解为 $a_i+k-1990$. 这种 $A_0$ 与 $A_k$ 间的对应是一一的.\n所以有\n$$\n\\left|A_0\\right|=\\left|A_1\\right|=\\left|A_2\\right|=\\left|A_3\\right|=\\left|A_4\\right|,\n$$\n于是\n$$\n\\begin{aligned}\n\\left|A_0\\right| & =\\frac{1}{5}\\left(\\left|A_0\\right|+\\left|A_1\\right|+\\left|A_2\\right|+\\left|A_3\\right|+\\left|A_4\\right|\\right) \\\\\n& =\\frac{1}{5}|M|=\\frac{1}{5} C_{1990}^{31} .\n\\end{aligned}\n$$\n说明在这里,我们的目的并不是求 $|M|$, 而是由于 $|M|$ 易于计算, 我们反过来利用这一点来达到计算 $\\left|A_0\\right|$ 的目的.",
|
6 |
+
"remark": "",
|
7 |
+
"figures": []
|
8 |
+
}
|
processed_dataset/calculation/0030.json
ADDED
@@ -0,0 +1,8 @@
|
|
|
|
|
|
|
|
|
|
|
|
|
|
|
|
|
|
|
1 |
+
{
|
2 |
+
"source_file": "./raw_volume-zh/volume1/chapter9.tex",
|
3 |
+
"problem_type": "calculation",
|
4 |
+
"problem": "例3. 设集合 $S=\\{1,2, \\cdots, 1000\\}, A$ 是 $S$ 的子集,且 $A$ 的元素或是 3 的倍数, 或是 7 的倍数.\n试求 $A$ 的元素个数的最大值.",
|
5 |
+
"solution": "解:设 $A_1=\\{x \\mid x \\in S$, 且 $3 \\mid x\\}, A_2=\\{x \\mid x \\in S$, 且 $7 \\mid x\\}$, 则 $|A|_{\\text {max }}=\\left|A_1 \\cup A_2\\right|$. 显然有\n$$\n\\begin{gathered}\n\\left|A_1\\right|=\\left[\\frac{1000}{3}\\right]=333, \\\\\n\\left|A_2\\right|=\\left[\\frac{1000}{7}\\right]=142, \\\\\n\\left|A_1 \\cap A_2\\right|=\\left[\\frac{1000}{3 \\cdot 7}\\right]=47 .\n\\end{gathered}\n$$\n所以\n$$\n\\begin{aligned}\n\\left|A_1 \\cup A_2\\right| & =\\left|A_1\\right|+\\left|A_2\\right|-\\left|A_1 \\cap A_2\\right| \\\\\n& =333+142-47=428 .\n\\end{aligned}\n$$\n所以 $A$ 的元素个数的最大值为 428 .\n说明利用容斥原理的关键是构造所要计数的集合的一个合适的覆盖.\n上例解答中的覆盖是: $A_1, A_2$.",
|
6 |
+
"remark": "",
|
7 |
+
"figures": []
|
8 |
+
}
|
processed_dataset/calculation/0031.json
ADDED
@@ -0,0 +1,8 @@
|
|
|
|
|
|
|
|
|
|
|
|
|
|
|
|
|
|
|
1 |
+
{
|
2 |
+
"source_file": "./raw_volume-zh/volume1/chapter9.tex",
|
3 |
+
"problem_type": "calculation",
|
4 |
+
"problem": "例5. 设 $S$ 是有理数 $r$ 的集合, 其中 $0<r<1$, 且 $r$ 有循环小数的展开形式为 $\\overline{0 . a b c a b c a b c \\cdots}=\\overline{0 .\\dot{a} b \\dot{c}}, a 、 b 、 c$ 不一定相异.\n在 $S$ 的元素中, 能写成最简分数的不同的分子有多少个?",
|
5 |
+
"solution": "解:因为 $\\overline{0 . \\dot{a} b \\dot{c}}=\\frac{\\overline{a b c}}{999}$, 又 $999=3^3 \\cdot 37$, 故如果 $\\overline{a b c}$ 既不能被 3 整除也不能被 37 整除,则分数就是最简形式.\n设 $A_1=\\{$ 不超过 1000 的正整数中 3 的倍数 $\\}, A_2=$ \\{不超过 1000 的正整数中 37 的倍数 $\\}$. 易知\n$$\n\\begin{gathered}\n\\left|A_1\\right|=\\frac{999}{3}=333,\\left|A_2\\right|=\\frac{999}{37}=27, \\\\\n\\left|A_1 \\cap A_2\\right|=\\frac{999}{3 \\cdot 37}=9 .\n\\end{gathered}\n$$\n由定理 2 , 有\n$$\n\\begin{aligned}\n& 999-\\left(\\left|A_1\\right|+\\left|A_2\\right|\\right)+\\left|A_1 \\cap A_2\\right| \\\\\n= & 999-(333+27)+9=648 .\n\\end{aligned}\n$$\n即此类最简分数的不同分子有 648 个.\n此外, 还有形如 $\\frac{k}{37}$ 的数, 其中自然数 $k$ 是小于 37 的 3 的倍数, 这样的 $k$ 有 $3,6,9, \\cdots, 36$ 共 12 个.\n故满足条件的分子有 $648+12=660$ 个.",
|
6 |
+
"remark": "",
|
7 |
+
"figures": []
|
8 |
+
}
|
processed_dataset/calculation/0032.json
ADDED
@@ -0,0 +1,8 @@
|
|
|
|
|
|
|
|
|
|
|
|
|
|
|
|
|
|
|
1 |
+
{
|
2 |
+
"source_file": "./raw_volume-zh/volume1/chapter9.tex",
|
3 |
+
"problem_type": "calculation",
|
4 |
+
"problem": "例6. 对于任何的集合 $S$, 设 $n(S)$ 为集合 $S$ 的子集个数.\n如果 $A 、 B 、 C$ 是三个集合,满足下列条件:\n(1) $n(A)+n(B)+n(C)=n(A \\cup B \\cup C)$,\n(2) $|A|=|B|=100$,\n求 $|A \\cap B \\cap C|$ 的最小值.",
|
5 |
+
"solution": "解:如果一个集合有 $k$ 个元素, 那么它有 $2^k$ 个子集.\n由题设有\n$$\n2^{100}+2^{100}+2^{|C|}=2^{|A \\cup B \\cup C|},\n$$\n即\n$$\n1+2^{|C|-101}=2^{|A \\cup B \\cup C|-101} .\n$$\n因为 $1+2^{|C|-101}$ 是大于 1 且等于一个 2 的整数幂, 所以 $|C|=101$. 从而有\n$$\n|A \\cup B \\cup C|=102 \\text {. }\n$$\n由容斥原理得\n$$\n\\begin{aligned}\n|A \\cap B \\cap C|= & |A \\cup B \\cup C|+|A|+|B|+|C| \\\\\n& -|A \\cup B|-|A \\cup C|-|B \\cup C| .\n\\end{aligned}\n$$\n从而有\n$$\n|A \\cap B \\cap C|=403-|A \\cup B|-|A \\cup C|-|B \\cup C| \\text {. }\n$$\n由 $A \\cup B 、 A \\cup C 、 B \\cup C \\subseteq A \\cup B \\cup C$ 得, $|A \\cup B| 、|A \\cup C|$ 、 $|B \\cup C| \\leqslant 102$, 所以\n$$\n|A \\cap B \\cap C| \\geqslant 403-102 \\times 3=97 .\n$$\n另一方面, 取 $A=\\{1,2,3, \\cdots, 100\\}, B=\\{3,4,5, \\cdots, 102\\}, C=\\{1,2,4,5,6, \\cdots, 100,101,102\\}$, 满足题设条件.\n这时\n$$\n|A \\cap B \\cap C|=|\\{4,5,6, \\cdots, 100\\}|=97 .\n$$\n所以, $|A \\cap B \\cap C|$ 的最小值为 97 .",
|
6 |
+
"remark": "",
|
7 |
+
"figures": []
|
8 |
+
}
|
processed_dataset/calculation/0033.json
ADDED
@@ -0,0 +1,8 @@
|
|
|
|
|
|
|
|
|
|
|
|
|
|
|
|
|
|
|
1 |
+
{
|
2 |
+
"source_file": "./raw_volume-zh/volume1/chapter9.tex",
|
3 |
+
"problem_type": "calculation",
|
4 |
+
"problem": "例8. 将与 105 互质的所有正整数从小到大排成数列, 求这个数列的第 1000 项.",
|
5 |
+
"solution": "分析:先看在区间 $(0,105]$ 中有多少个整数与 105 互质.\n因为 $105=3 \\times 5 \\times 7$, 所以只要在数列 $1,2, \\cdots, 105$ 中去掉所有 3 或 5 或 7 的倍数即可.\n然后再逐段考察区间 $(105 \\cdot(k-1), 105 k]$ 中与 105 互质的整数.\n解设 $S=\\{1,2, \\cdots, 105\\}, A_3=\\{a \\mid a \\in S$, 且 $3 \\mid a\\}, A_5=\\{a \\mid a \\in S$, 且 $5 \\mid a\\}, A_7=\\{a \\mid a \\in S$, 且 $7 \\mid a\\}$, 则\n$$\n\\begin{aligned}\n& \\left|A_3\\right|=\\frac{105}{3}=35,\\left|A_5\\right|=\\frac{105}{5}=21,\\left|A_7\\right|=\\frac{105}{7}=15, \\\\\n& \\left|A_3 \\cap A_5\\right|=\\frac{105}{3 \\times 5}=7,\\left|A_5 \\cap A_7\\right|=\\frac{105}{5 \\times 7}=3, \\\\\n& \\left|A_7 \\cap A_3\\right|=\\frac{105}{7 \\times 3}=5, \\\\\n& \\left|A_3 \\cap A_5 \\cap A_7\\right|=\\frac{105}{3 \\times 5 \\times 7}=1,|S|=105 .\n\\end{aligned}\n$$\n在 1 到 105 中,与 105 互质的数有\n$$\n\\begin{aligned}\n& \\left|\\complement_S A_3 \\cap \\complement_S A_5 \\cap \\complement_S A_7\\right| \\\\\n= & |S|-\\left|A_3 \\cup A_5 \\cup A_7\\right| \\\\\n= & |S|-\\left(\\left|A_3\\right|+\\left|A_5\\right|+\\left|A_7\\right|\\right) \\\\\n& +\\left(\\left|A_3 \\cap A_5\\right|+\\left|A_5 \\cap A_7\\right|\\right. \\\\\n& \\left.+\\left|A_7 \\cap A_3\\right|\\right)-\\left|A_3 \\cap A_5 \\cap A_7\\right| \\\\\n= & 105-(35+21+15)+(7+3+5)-1 \\\\\n= & 48 .\n\\end{aligned}\n$$\n设与 105 互质的正整数按从小到大的顺序排列为 $a_1, a_2, \\cdots, a_n, \\cdots$, 则\n$$\n\\begin{gathered}\na_1=1, a_2=2, a_3=4, \\cdots, a_{48}=104, \\\\\na_{49}=105+1, a_{50}=105+2, \\\\\na_{51}=105+4, \\cdots, a_{96}=105+104, \\cdots\n\\end{gathered}\n$$\n因为 $1000=48 \\times 20+40$, 所以\n$$\na_{1000}=105 \\times 20+a_{40} .\n$$\n由于 $a_{48}=104, a_{47}=103, a_{46}=101, a_{45}=97, a_{44}=94, a_{43}=92$, $a_{42}=89, a_{41}=88, a_{40}=86$, 所以\n$$\na_{1000}=105 \\times 20+86=2186 .\n$$\n筛法公式在数论中的一个典型应用, 就是推导欧拉函数的解析式.\n我们把不超过正整数 $n$ 且与 $n$ 互质的正整数的数目记为 $\\varphi(n)$, 称为欧拉函数.\n例如, $\\varphi(2)=1, \\varphi(3)=2, \\varphi(6)=2, \\varphi(8)=4$.",
|
6 |
+
"remark": "",
|
7 |
+
"figures": []
|
8 |
+
}
|
processed_dataset/calculation/0034.json
ADDED
@@ -0,0 +1,8 @@
|
|
|
|
|
|
|
|
|
|
|
|
|
|
|
|
|
|
|
1 |
+
{
|
2 |
+
"source_file": "./raw_volume-zh/volume1/exercise1.tex",
|
3 |
+
"problem_type": "calculation",
|
4 |
+
"problem": "问题1: 已知三元实数集 $A=\\{x, x y, x+y\\}, B=\\{0,|x|, y\\}$, 且 $A=B$, 则$x^{2005}+y^{2005}=$",
|
5 |
+
"solution": "解: 0 .",
|
6 |
+
"remark": "",
|
7 |
+
"figures": []
|
8 |
+
}
|
processed_dataset/calculation/0035.json
ADDED
@@ -0,0 +1,8 @@
|
|
|
|
|
|
|
|
|
|
|
|
|
|
|
|
|
|
|
1 |
+
{
|
2 |
+
"source_file": "./raw_volume-zh/volume1/exercise1.tex",
|
3 |
+
"problem_type": "calculation",
|
4 |
+
"problem": "问题2: 设集合 $S=\\left\\{(x, y) \\mid x-\\frac{1}{2} y^2\\right.$ 为奇数, $\\left.x, y \\in \\mathbf{R}\\right\\}, T=\\{(x, y) \\left.\\sin (2 \\pi x)-\\sin \\left(\\pi y^2\\right)=\\cos (2 \\pi x)-\\cos \\left(\\pi y^2\\right), x, y \\in \\mathbf{R}\\right\\}$. 则 $S$ 与 $T$ 的关系是",
|
5 |
+
"solution": "解: $S \\varsubsetneqq T$. 当 $x=\\frac{1}{2} y^2+$ 奇数时, 显然 $\\sin (2 \\pi x)-\\sin \\left(\\pi y^2\\right)=\\cos (2 \\pi x)-\\cos \\left(\\pi y^2\\right)$ 成立, $S \\subseteq T$. 但满足 $x=\\frac{1}{2} y^2$ 的点 $(x, y) \\in T$, 而不属于 $S$, 故 $S \\varsubsetneqq T$.",
|
6 |
+
"remark": "",
|
7 |
+
"figures": []
|
8 |
+
}
|
processed_dataset/calculation/0036.json
ADDED
@@ -0,0 +1,8 @@
|
|
|
|
|
|
|
|
|
|
|
|
|
|
|
|
|
|
|
1 |
+
{
|
2 |
+
"source_file": "./raw_volume-zh/volume1/exercise1.tex",
|
3 |
+
"problem_type": "calculation",
|
4 |
+
"problem": "问题3: 集合 $M=\\{u \\mid u=12 m+8 n+4 l, m, n, l \\in \\mathbf{Z}\\}$ 与 $N=\\{u \\mid u=20 p+ 16 q+12 r, p, q, r \\in \\mathbf{Z}\\}$ 的关系为",
|
5 |
+
"solution": "解: $M=N$. 因 $12 m+8 n+4 l=4(3 m+2 n+l), 20 p+16 q+12 r=4(5 p+4 q+3 r),(3,2,1)=1,(5,4,3)=1$, 由裴蜀定理可知 $3 m+2 n+ l$ 与 $5 p+4 q+3 r$ 均可表示所有整数.\n所以, $M=N=\\{k \\mid k=4 t, t \\in \\mathbf{Z}\\}$.",
|
6 |
+
"remark": "",
|
7 |
+
"figures": []
|
8 |
+
}
|
processed_dataset/calculation/0037.json
ADDED
@@ -0,0 +1,10 @@
|
|
|
|
|
|
|
|
|
|
|
|
|
|
|
|
|
|
|
|
|
|
|
1 |
+
{
|
2 |
+
"source_file": "./raw_volume-zh/volume1/exercise1.tex",
|
3 |
+
"problem_type": "calculation",
|
4 |
+
"problem": "问题4: 设 $A=\\{(x, y) \\mid 0 \\leqslant x \\leqslant 2,0 \\leqslant y \\leqslant 2\\}, B=\\{(x, y) \\mid x \\leqslant 10, y \\geqslant 2, y \\leqslant x-4\\}$ 是直角坐标平面 $x O y$ 上的点集.\n则 $C=\\left\\{frac{x_1+x_2}{2}\\frac{y_1+y_2}{2}\\right\\} \\mid\\left(x_1, y_1\\right) \\in A,\\left(x_2, y_2\\right) \\in B\\right\\}$ 所成图形的面积是",
|
5 |
+
"solution": "解: 7. 如图(<FilePath:./images/volume1/figures/fig-c1p4.png>),集合 $A$ 为正方形 $O A B C$, 集合 $B$ 为 Rt $\\triangle D E F . O D 、 A E 、 B F 、 C F$ 、 $C D$ 的中点依次为 $M(3,1) 、 N(6,1)$ 、 $P(6,4) 、 Q(5,4) 、 R(3,2)$. 所成图形面积 $S_{M N P Q R}=7$.",
|
6 |
+
"remark": "",
|
7 |
+
"figures": [
|
8 |
+
"./images/volume1/figures/fig-c1p4.png"
|
9 |
+
]
|
10 |
+
}
|
processed_dataset/calculation/0038.json
ADDED
@@ -0,0 +1,8 @@
|
|
|
|
|
|
|
|
|
|
|
|
|
|
|
|
|
|
|
1 |
+
{
|
2 |
+
"source_file": "./raw_volume-zh/volume1/exercise1.tex",
|
3 |
+
"problem_type": "calculation",
|
4 |
+
"problem": "问题5: 已知非空数集 $M \\subseteq\\{1,2,3,4,5\\}$, 则满足条件“若 $x \\in M$, 则 $6-x \\in M$ ” 的集合 $M$ 的个数是",
|
5 |
+
"solution": "解: 7. 因为 $1+5=2+4=3+3$, 故 $M$可以是 $\\{3\\},\\{1,5\\},\\{2,4\\},\\{1,3,5\\},\\{2,3,4\\},\\{1,2,4,5\\},\\{1,2,3$, $4,5\\}$.",
|
6 |
+
"remark": "",
|
7 |
+
"figures": []
|
8 |
+
}
|
processed_dataset/calculation/0039.json
ADDED
@@ -0,0 +1,8 @@
|
|
|
|
|
|
|
|
|
|
|
|
|
|
|
|
|
|
|
1 |
+
{
|
2 |
+
"source_file": "./raw_volume-zh/volume1/exercise1.tex",
|
3 |
+
"problem_type": "calculation",
|
4 |
+
"problem": "问题6: 设 $a \\in \\mathbf{R}^{+}, A=\\left\\{(x, y) \\mid(x-1)^2+(y-2)^2 \\leqslant \\frac{4}{5}\\right\\}$ 与 $B=\\{(x, y) \\mid |x-1|+2|y-2| \\leqslant a\\}$ 是直角坐标平面 $x O y$ 内的点集.\n则 $A \\subseteq B$ 的充要条件是",
|
5 |
+
"solution": "解: $a \\geqslant 2$. 集合 $A$ 为以 $(1,2)$ 为圆心 $、 \\frac{2}{\\sqrt{5}}$ 为半径的圆面.\n集合 $B$ 为以 $(1,2)$ 为对角线交点的菱形, 且平行于 $x$ 轴的对角线长为 $2 a$, 平行于 $y$ 轴的对角线长为 $a$. 由 $A \\subseteq B$ 知, 当 $a \\cdot \\frac{a}{2}=\\frac{2}{\\sqrt{5}} \\cdot \\sqrt{a^2+\\left(\\frac{a}{2}\\right)^2}$ 时 $a$ 值最小, 所以 $a_{\\min }=2$.",
|
6 |
+
"remark": "",
|
7 |
+
"figures": []
|
8 |
+
}
|
processed_dataset/calculation/0040.json
ADDED
@@ -0,0 +1,8 @@
|
|
|
|
|
|
|
|
|
|
|
|
|
|
|
|
|
|
|
1 |
+
{
|
2 |
+
"source_file": "./raw_volume-zh/volume1/exercise1.tex",
|
3 |
+
"problem_type": "calculation",
|
4 |
+
"problem": "问题7: 集合 $\\left\\{x \\mid-1 \\leqslant \\log _{\\frac{1}{x}} 10<-\\frac{1}{2}, x>1\\right.$ 且 $\\left.x \\in \\mathbf{N}\\right\\}$ 的真子集的个数是",
|
5 |
+
"solution": "解: $2^{90}-1$.",
|
6 |
+
"remark": "",
|
7 |
+
"figures": []
|
8 |
+
}
|
processed_dataset/calculation/0041.json
ADDED
@@ -0,0 +1,8 @@
|
|
|
|
|
|
|
|
|
|
|
|
|
|
|
|
|
|
|
1 |
+
{
|
2 |
+
"source_file": "./raw_volume-zh/volume1/exercise1.tex",
|
3 |
+
"problem_type": "calculation",
|
4 |
+
"problem": "问题8: 已知 $A=\\left\\{x \\mid x^2-4 x+3<0, x \\in \\mathbf{R}\\right\\}, B=\\left\\{x \\mid 2^{1-x}+a \\leqslant 0, x^2-\\right. 2(a+7) x+5 \\leqslant 0, x \\in \\mathbf{R}\\}$. 若 $A \\subseteq B$, 则实数 $a$ 的取值范围是",
|
5 |
+
"solution": "解: $-4 \\leqslant a \\leqslant-1$. 易知 $A=(1,3)$. 记 $f(x)=2^{1-x}+a, g(x)=x^2-2(a+7) x+5 . A \\subseteq B$ 表明, 当 $1<x<3$ 时, 函数 $f(x)$ 与 $g(x)$ 的图象都在$x$ 轴的下方.\n$A \\subseteq B$ 的充要条件是: $f(1) \\leqslant 0, g(1) \\leqslant 0$ 和 $f(3) \\leqslant 0, g(3) \\leqslant 0$ 同时成立.\n解之即得.",
|
6 |
+
"remark": "",
|
7 |
+
"figures": []
|
8 |
+
}
|
processed_dataset/calculation/0042.json
ADDED
@@ -0,0 +1,8 @@
|
|
|
|
|
|
|
|
|
|
|
|
|
|
|
|
|
|
|
1 |
+
{
|
2 |
+
"source_file": "./raw_volume-zh/volume1/exercise1.tex",
|
3 |
+
"problem_type": "calculation",
|
4 |
+
"problem": "问题9: 已知 $M=\\left\\{x \\mid x=a^2+1, a \\in \\mathbf{N}^*\\right\\}, N=\\left\\{x \\mid x=b^2-4 b+5, b \\in \\mathbf{N}^*\\right\\}$, 则 $M$ 与 $N$ 的关系是",
|
5 |
+
"solution": "解: $M \\varsubsetneqq N$. 由 $a^2+1=(a+2)^2-4 a+5$ 知 $M \\subseteq N$. 但 $1 \\in N, 1 \\notin M$.",
|
6 |
+
"remark": "",
|
7 |
+
"figures": []
|
8 |
+
}
|
processed_dataset/calculation/0043.json
ADDED
@@ -0,0 +1,8 @@
|
|
|
|
|
|
|
|
|
|
|
|
|
|
|
|
|
|
|
1 |
+
{
|
2 |
+
"source_file": "./raw_volume-zh/volume1/exercise1.tex",
|
3 |
+
"problem_type": "calculation",
|
4 |
+
"problem": "问题10: 非空集合 $S$ 满足:\n(1) $S \\subseteq\\{1,2, \\cdots, 2 n+1\\}, n \\in \\mathbf{N}^*$;\n(2) 若 $a \\in S$, 则有 $2 n+2-a \\in S$.\n那么, 同时满足 (1)、(2) 的非空集合 $S$ 的个数是",
|
5 |
+
"solution": "解: $2^{n+1}-1$. 把自然数 $1,2, \\cdots, 2 n+1$ 搭配成 $n+1$ 个数组 $\\{1,2 n+1\\}, \\{2,2 n\\}, \\cdots,\\{n, n+2\\},\\{n+1\\} . S$ 的元素从以上 $n+1$ 组选取, 有 $\\mathrm{C}_{n+1}^1+ \\mathrm{C}_{n+1}^2+\\cdots+\\mathrm{C}_{n+1}^{n+1}=2^{n+1}-1$ 种取法.",
|
6 |
+
"remark": "",
|
7 |
+
"figures": []
|
8 |
+
}
|
processed_dataset/calculation/0044.json
ADDED
@@ -0,0 +1,8 @@
|
|
|
|
|
|
|
|
|
|
|
|
|
|
|
|
|
|
|
1 |
+
{
|
2 |
+
"source_file": "./raw_volume-zh/volume1/exercise1.tex",
|
3 |
+
"problem_type": "calculation",
|
4 |
+
"problem": "问题11: 设由模为 1 的 $n(2<n<6)$ 个复数组成的集合 $S$ 满足下面两条:\n(1) $1 \\in S$;\n(2) 若 $z_1 \\in S, z_2 \\in S$, 则 $z_1-2 z_2 \\cos \\theta \\in S$, 其中 $\\theta=\\arg \\frac{z_1}{z_2}$.\n则集合 $S=$",
|
5 |
+
"solution": "解: $\\{1,-1, \\mathrm{i},-\\mathrm{i}\\}$. 当 $z_1=z_2=z$ 时, 若 $z \\in S$, 则 $z_1-2 z_2 \\cos \\theta=-z \\in S$. 因 $|z|=1$, 所以 $|z|=|-z|=1$. 这说明 $S$ 中含有偶数个元素.\n又 $2<n<6$, 所以 $n=4$. 由 $1 \\in S$, 得 $-1 \\in S$. 设 $z_1=\\cos \\alpha+i \\sin \\alpha(\\sin \\alpha \\neq 0$, $0 \\leqslant \\alpha<2 \\pi), z_2=1, \\theta=\\arg \\left(\\frac{z_1}{z_2}\\right)=\\alpha$. 若 $z_1 \\in S$, 则 $z_1-2 \\cos \\theta=-\\cos \\alpha+i \\sin \\alpha \\in S$. 因为 $\\sin \\alpha \\neq 0$, 故 $\\cos \\alpha+i \\sin \\alpha \\neq-(\\cos \\alpha+i \\sin \\alpha)$, 所以 $\\cos \\alpha+\\mathrm{i} \\sin \\alpha=-\\cos \\alpha+\\mathrm{i} \\sin \\alpha$, 即 $\\cos \\alpha=0, \\sin \\alpha= \\pm 1$. 所以 $i \\in S,-i \\in S$.",
|
6 |
+
"remark": "",
|
7 |
+
"figures": []
|
8 |
+
}
|
processed_dataset/calculation/0045.json
ADDED
@@ -0,0 +1,8 @@
|
|
|
|
|
|
|
|
|
|
|
|
|
|
|
|
|
|
|
1 |
+
{
|
2 |
+
"source_file": "./raw_volume-zh/volume1/exercise1.tex",
|
3 |
+
"problem_type": "calculation",
|
4 |
+
"problem": "问题12: 集合 $A=\\left\\{x_1, x_2, x_3, x_4, x_5\\right\\}$,计算 $A$ 中的二元子集两元素之和组成集合 $B=\\{3,4,5,6,7,8,9,10,11,13\\}$. 则 $A=$",
|
5 |
+
"solution": "解: $\\{1,2,3,5,8\\}$. 不妨设 $x_1<x_2<x_3<x_4<x_5$. 则 $x_1+x_2=3$, $x_4+x_5=13$. 又 $4\\left(x_1+x_2+x_3+x_4+x_5\\right)=3+4+5+6+7+8+9+10+11+13$, 即 $x_1+x_2+x_3+x_4+x_5=19$, 从而得 $x_3=19-3-13=3$. 又 $x_1+ x_3=4$, 从而 $x_1=1$. 又 $x_3+x_5=11$, 从而 $x_5=8$, 所以 $x_2=2, x_4=5$.",
|
6 |
+
"remark": "",
|
7 |
+
"figures": []
|
8 |
+
}
|
processed_dataset/calculation/0046.json
ADDED
@@ -0,0 +1,8 @@
|
|
|
|
|
|
|
|
|
|
|
|
|
|
|
|
|
|
|
1 |
+
{
|
2 |
+
"source_file": "./raw_volume-zh/volume1/exercise1.tex",
|
3 |
+
"problem_type": "calculation",
|
4 |
+
"problem": "问题14: 称有限集 $S$ 的所有元素的乘积为 $S$ 的 “积数”, 给定数集 $M= \\left\\{\\frac{1}{2}, \\frac{1}{3}, \\cdots, \\frac{1}{100}\\right\\}$. 求集 $M$ 的所有含偶数个元素的子集的“积数”之和.",
|
5 |
+
"solution": "解: 设集合 $M$ 的所有含偶数个元数的子集的积数之和为 $x$, 所有含奇数个元素的子集的积数之和为 $y$, 则 $x+y=\\left(1+\\frac{1}{2}\\right)\\left(1+\\frac{1}{3}\\right) \\cdots\\left(1+\\frac{1}{100}\\right)-1, x-y=\\left(1-\\frac{1}{2}\\right)\\left(1-\\frac{1}{3}\\right) \\cdots\\left(1-\\frac{1}{100}\\right)-1$. 所以 $x+y=\\frac{99}{2}, x-y= -\\frac{99}{100}$. 解得 $x=\\frac{4851}{200}$.",
|
6 |
+
"remark": "",
|
7 |
+
"figures": []
|
8 |
+
}
|
processed_dataset/calculation/0047.json
ADDED
@@ -0,0 +1,8 @@
|
|
|
|
|
|
|
|
|
|
|
|
|
|
|
|
|
|
|
1 |
+
{
|
2 |
+
"source_file": "./raw_volume-zh/volume1/exercise1.tex",
|
3 |
+
"problem_type": "calculation",
|
4 |
+
"problem": "问题17: 设集合 $M=\\{1,2,3, \\cdots, 1000\\}$, 现对 $M$ 的任一非空子集 $X$,令 $\\alpha_X$ 表示 $X$ 中最大数与最小数之和.\n求所有这样的 $\\alpha_X$ 的算术平均值.",
|
5 |
+
"solution": "解: 构造子集 $X^{\\prime}=\\{1001-x \\mid x \\in X\\}$, 则所有非空子集分成两类 $X^{\\prime}=X$ 和 $X^{\\prime} \\neq X$. 当 $X^{\\prime}=X$ 时,必有 $X^{\\prime}=X=M$, 于是, $\\alpha_X=1001$. 当 $X^{\\prime} \\neq X$ 时, 设 $x 、 y$ 分别是 $X$ 中的最大数与最小数, 则 $1001-x 、 1001-y$ 分别是 $X^{\\prime}$ 中的最小数与最大数.\n于是, $\\alpha_X=x+y, \\alpha_{X^{\\prime}}=2002-x-y$. 从而, $\\frac{\\alpha_X+\\alpha_{X^{\\prime}}}{2}=1001$. 因此,所有的 $\\alpha_X$ 的算术平均值为 1001 .",
|
6 |
+
"remark": "",
|
7 |
+
"figures": []
|
8 |
+
}
|
processed_dataset/calculation/0048.json
ADDED
@@ -0,0 +1,8 @@
|
|
|
|
|
|
|
|
|
|
|
|
|
|
|
|
|
|
|
1 |
+
{
|
2 |
+
"source_file": "./raw_volume-zh/volume1/exercise2.tex",
|
3 |
+
"problem_type": "calculation",
|
4 |
+
"problem": "问题1 已知集合 $M=\\{2,|a|\\}$ 是全集 $U=\\left\\{2,3, a^2+2 a+2\\right\\}$ 的一个子集, 且 $\\complement_U M=\\{5\\}$, 则实数 $a$ 的值等于",
|
5 |
+
"solution": "-3 . $|a|=3$, 且 $a^2+2 a+2=5$. 解得 $a=-3$.",
|
6 |
+
"remark": "",
|
7 |
+
"figures": []
|
8 |
+
}
|
processed_dataset/calculation/0049.json
ADDED
@@ -0,0 +1,8 @@
|
|
|
|
|
|
|
|
|
|
|
|
|
|
|
|
|
|
|
1 |
+
{
|
2 |
+
"source_file": "./raw_volume-zh/volume1/exercise2.tex",
|
3 |
+
"problem_type": "calculation",
|
4 |
+
"problem": "问题2 已知全集 $I=\\mathbf{Z}, M=\\{x \\mid x=2 n, n \\in \\mathbf{Z}\\}, S=\\{x \\mid x=3 n, n \\in \\mathbf{Z}\\}$, 则 $M \\cap \\complement_{\\mathrm{z}} S=$",
|
5 |
+
"solution": "$\\{x \\mid x=6 n \\pm 2, n \\in \\mathbf{Z}\\} . M=\\{x \\mid x=6 n, 6 n+2,6 n+4, n \\in \\mathbf{Z}\\}$, $S=\\{x \\mid x=6 n, 6 n+3, n \\in \\mathbf{Z}\\}$, 于是 $M \\cap \\complement_{\\mathbf{Z}} S=\\{x \\mid x=6 n \\pm 2, n \\in \\mathbf{Z}\\}$.",
|
6 |
+
"remark": "",
|
7 |
+
"figures": []
|
8 |
+
}
|